Vous êtes sur la page 1sur 53

SOLUCIONES Y PROPIEDADES COLIGATIVAS

EXIJA SU LIBRO

3.1 DEFINICIÓN DE SOLUCIÓN

M uy pocos materiales que encontramos en la vida diaria son sustancias puras; la mayor
parte son mezclas y muchas de ellas son homogéneas, Recordemos que las mezclas
homogéneas se denominan soluciones o disoluciones, por ejemplo: el aire que respiramos es
una mezcla homogénea de varias sustancias gaseosas. El latón es una solución de cinc en
cobre. Los océanos son una solución de muchas sustancias que se han disuelto en agua.

Una solución es una mezcla homogénea de dos o mas sustancias puras,


denominadas componentes de la solución, las cuales pueden ser gaseosas,
líquidas o sólidas; debido a que las soluciones líquidas son las mas comunes, en esta sección
enfocaremos nuestro estudio en dicho estado. Sin embargo, el estado físico de una solución lo
determina a menudo el estado de su componente principal, denominado disolvente. El otro u
otros componentes se denominan soluto.

Soluto es la sustancia que se


Disolvente o solvente, es la sustancia
disuelve a través del disolvente en
donde se disuelve el llamado soluto.
forma de moléculas o iones para
Generalmente es aquella que se
formar una sola fase. En una
encuentra en mayor cantidad. En toda
solución puede existir mas de un
solución existe un solo solvente.
soluto.

AGUA
YUPI

COMPENDIOS DE QUÍMICA GENERAL PARA ESTUDIANTES DE CIENCIAS E INGENIERÍA


58 SOLUCIONES Y PROPIEDADES COLIGATIVAS
En la figura 3.1 es un resumen de la relación entre los materiales (materia), recuerde que las
sustancias pueden ser mezcladas por procesos físicos y se pueden usar otros procesos físicos
para separar la mezcla en 1sustancias.

3.2 PSEUDO - SOLUCIÓN, SUSPENSIÓN Y materia


COLOIDE

C uando una sustancia se disuelve o dispersa a


través de otra, formaremos una mezcla donde
encontramos tres posibilidades diferentes de
sustancias

Mezclas
mezclas

tamaños de partículas. Cada uno de estos casos elementos homogéneas


(soluciones)
dará lugar a mezclas denominadas: solución,
coloide y suspensión, con propiedades diferentes. Mezclas
compuestos heterogéneas
La tabla 4.1 muestra una clasificación según el
tamaño partículas.
Figura 3.1
Relación entre los materiales

1. Cuando la sustancia se disuelve en forma de moléculas o iones entonces se denomina


solución y los componentes son denominados soluto y disolvente.
2. Cuando la sustancia se dispersa (no es soluble) y permanece firmemente dividida se
denomina coloide y sus componentes son llamados fase dispersa y fase dispersante.
3. Cuando el tamaño de la sustancia a quién se le denomina realmente micela, es mas
grande en comparación a los dos casos anteriores se denominará suspensión.
TABLA 3.1
Clasificación de la mezclas según el tamaño de miscela

SOLUCION COLOIDE SUSPENSIÓN

TAMAÑO DE
MISCELAS 1-10Å 10-10000Å >10000Å

3.2.1 Coloides hidrofílicos e hidrofóbicos

C oloide es una pseudo – solución, es una mezcla heterogénea en la cual las partículas del
soluto no precipitan, se dispersan a través de una fase dispersante, se clasifican en dos
clases principales: coloides hidrofílicos y coloides hidrofóbicos. Un coloide hidrofílico (que
aman el agua) es un coloide en el cual hay una atracción fuerte entre la fase dispersa y la fase
continua (agua). Muchos de estos coloides consisten en macromoléculas (moléculas muy
grandes) dispersas en agua. Excepto por el gran tamaño de las moléculas dispersas, estos
coloides parecen soluciones normales. Un coloide hidrofóbico (que rechazan el agua), es un
coloide en el cual hay una falta de atracción entre la fase dispersa y la fase continua (agua)
Los coloides hidrofóbicos son básicamente inestables.

1
1 Ebbing Darrell D. Según este autor una sustancia (o sustancia pura) es una clase de materia que no puede ser
separada en otras clases de materia por ningún proceso físico.

COMPENDIOS DE QUÍMICA GENERAL PARA ESTUDIANTES DE CIENCIAS E INGENIERÍA


SOLUCIONES Y PROPIEDADES COLIGATIVAS 59
La tabla 3.2 muestra una clasificación de 8 tipos de coloides que son producto de la mezcla
entre los estados sólido, líquido y gaseoso, pero respetando el orden para identificar a la fase
dispersa y fase dispersante. No se incluye la mezcla gas-gas porque estos se componen de
moléculas individuales.

TABLA 3.2
Clasificación de los coloides según el estado de agregación

FASE FASE TIPO DE EJEMPLO


DISPERSA DISPERSANTE COLOIDE
Gas Gas ................. ..................
Líquido Gas Aerosol líquido Niebla
Sólido Gas Aerosol sólido Humo
Gas Líquido Espuma Espuma de cerveza
Líquido Líquido Emulsión Leche
Sólido Líquido Sol y gel Pintura, jabón en agua
Gas Sólido Espuma sólida Esponja, piedra pómez
Líquido Sólido Emulsión sólida Mantequilla
Sólido Sólido Sol sólida Porcelana

Una de las formas prácticas que permite identificar a los coloides, es el hecho de aprovecha el
efecto Tyndall, donde se refleja la luz mucho mas intensa en una suspensión que en un
coloide, en una solución no hay dicho reflejo.

3.3 CLASIFICACIÓN DE LAS Haz de luz no visible


SOLUCIONES

L as soluciones se pueden clasificar,


atendiendo a 5 aspectos importantes:
Fuente luminosa

Haz de luz no
visible
A B C

3.3.1 Según el número de componentes

 Soluciones binarias: de dos


Haz de luz visible Haz de luz visible
componentes
Figura 3.2
 Soluciones ternarias: de tres Dispersión de un haz de luz por partículas
componentes coloidales conocido como efecto Tyndall. La
 Soluciones cuaternarias: de cuatro presencia de partículas coloidales pueden
componentes detectarse con facilidad con ayuda de un haz de
luz.
A: Oro coloidal, una solución dorada
3.3.2 Según la naturaleza del disolvente B: Solución de NaCl
C: Humo de tabaco, Un aerosol
 Soluciones acuosas: el disolvente
es el agua
 Soluciones orgánicas: El disolvente puede ser: benceno, acetona, etc

3.3.3 Según la naturaleza del soluto

 Soluciones ácidas: presencia de sustancias ácidas: HCl, HNO3, etc.


 Soluciones básicas: presencia de sustancias básicas: NaOH, Ca(OH)2
COMPENDIOS DE QUÍMICA GENERAL PARA ESTUDIANTES DE CIENCIAS E INGENIERÍA
60 SOLUCIONES Y PROPIEDADES COLIGATIVAS
 Soluciones neutras: presencia de sales neutras: NaCl, KNO3, etc.

3.3.4 De acuerdo a la cantidad de sus componentes

 Soluciones diluidas: Cuando la masa del soluto es pequeña con respecto al


volumen total de la solución.
 Soluciones concentradas: Es aquella donde la cantidad del soluto es grande
respecto al volumen total de la solución.
 Soluciones saturadas: Es aquella solución que ha alcanzado su máxima
concentración a una temperatura determinada, es decir que no admite mas soluto
porque este comienza a precipitar.
 Soluciones sobresaturadas: Es cuando se disuelve mas soluto que la solución
saturada debido a ciertos factores mecánicos, por ejemplo la agitación donde se
encuentra en equilibrio inestable.

3.3.5 Según los estados de agregación de la materia

 Soluciones sólidas: Donde sus componentes se hallan en el estado sólido.


 Soluciones liquidas: Donde sus componentes están estado líquido.
 Soluciones gaseosas: Donde sus componentes están en estado gaseoso.

Ejemplo 3.1.- Dar un ejemplo de una solución


constituida por las siguientes fases a) gas-gas, b)
líquido-gas, c) líquido-líquido, d) líquido-sólido, e)
sólido-sólido.
Solubilidad en g soluto/100 g agua

Solución:
a) Gas- gas : Aire (O2, CO2,
etc en N2)
b) Líquido – gas: Soda ( CO2 (g)
en H2O ())
c) Líquido – líquido: alcohol para
fricciones (H2O () en i-C3H7OH ())
d) Líquido – sólido: agua de mar
(NaCl, LiCl, etc en H2O ()) KI
200
e) Sólido-sólido: “oro” de 14
180
quilates (Cu en Au)
160 KNO3
3.4 SOLUBILIDAD 140

L
120
a solubilidad se define como la cantidad de una KBr
100
sustancia que se disuelve en una cantidad dada
de disolvente (por ejemplo agua) a una temperatura 80
KCl
dada para dar una solución saturada; es la 60
concentración máxima posible. Por ejemplo, la sal 40 NaCl
de cocina tiene una solubilidad de 36 g por 100 g de 20 K2SO4
agua; por lo tanto, al añadir 40 gramos de NaCl,
quedaran 3 g de sal sin disolverse. 10 20 30 40 50 60 70 80 90 100
Temperatura en grados Celcius

COMPENDIOS DE QUÍMICA GENERAL PARA ESTUDIANTES DE CIENCIAS E INGENIERÍA


Figura 3.3
Efecto de la temperatura en las solubilidades de algunas
sales.
SOLUCIONES Y PROPIEDADES COLIGATIVAS 61
3.4.1 Factores que afectan la solubilidad

L a solubilidad de un sólido es muy sensible a los cambios de temperatura; pero los


cambios ordinarios de presión no producen ninguna variación significativa. La figura 3.3
muestra cómo la temperatura afecta la solubilidad de algunos sólidos corrientes en agua, por
ejemplo la sacarosa, el nitrato de amonio y el hidróxido de potasio ilustran muy bien este
fenómeno; por el contrario, la solubilidad del NaCl casi no se ve afectada por un cambio de
temperatura. En algunos casos es posible lograr concentraciones superiores a las
concentraciones de saturación, y entonces se tiene una solución sobresaturada. Una solución
sobresaturada se puede producir enfriando una solución concentrada caliente.

Los compuestos iónicos son solubles en agua, alcohol metílico y amoniaco líquido; e
insolubles en octano, benceno y tetracloruro de carbono. Las moléculas de agua, alcohol
metílico y amoniaco son polares; cada molécula
δ- δ-
posee centros positivos y negativos de carga
O H O
δ+ δ +
δ+ eléctrica como muestra la figura 3.4. Otros
H H C H líquidos polares también actúan eficientemente
AGUA en la disolución de compuestos iónicos. Las
H
H moléculas de octano y benceno son no polares
N ALCOHOL METILICO (ver figura 3.5). Puesto que las
electronegatividades del carbono y del hidrógeno
H H es muy pequeña, cualquier enlace entre estos dos
H AMONIACO átomos es no polar. El enlace entre el cloro y el
carbono sí es polar, como se puede deducir al
Figura 3.4 estudiar las 2electronegatividades de la tabla 1.3
Algunas moléculas polares del capítulo 1 (página 20)

Por regla general los líquidos no polares son ineficientes en la disolución de compuestos
iónicos. También encontramos que un líquido polar disolverá otros compuestos polares. El
amoniaco y el alcohol metílico se disuelven en agua. La sacarosa, sólido polar (no iónico), es
soluble en agua y alcohol metílico, e insoluble en benceno, octano y tetracloruro de carbono.
Los disolventes no polares disuelven compuestos no polares. Las grasas y aceites,
hidrocarburos no polares, se disuelven en cualquiera de los tres solventes no polares que
hemos discutido previamente, pero son
virtualmente insolubles en los tres disolventes
polares. La regla que se sigue en la elección de H Cl
disolventes dice: “ sustancias similares C
disuelven sustancias similares”. Los H C C H C
disolventes polares disuelven compuestos
iónicos y polares; y los no polares disuelven H C C H Cl Cl
compuestos no polares. C Cl
H
3.4.2 Solubilidad de los gases TETRACLORURO
BENCENO DE CARBONO

L os gases son muy poco solubles en agua y


otros líquidos corrientes. La solubilidad de
un gas puede acrecentar aumentando la presión
Figura 3.5
Algunas moléculas no polares

2
La electronegatividad de un elemento mide la tendencia relativa del átomo a atraer los electrones hacia sí
cuando se combina químicamente con otro átomo ( véase capítulo 1 ).
COMPENDIOS DE QUÍMICA GENERAL PARA ESTUDIANTES DE CIENCIAS E INGENIERÍA
62 SOLUCIONES Y PROPIEDADES COLIGATIVAS
sobre el gas que se halla por encima de la solución. La solubilidad del oxígeno a diferentes
presiones aparece en la tabla 4.3. Es obvio que al observar la última columna, que la
solubilidad es directamente proporcional a la presión del gas. Este comportamiento es normal
para soluciones de gases en líquidos.

TABLA 3.3
Solubilidad del oxígeno en agua a 25 ºC
P [mmHg] Solubilidad, [moles O2 m La solubilidad de los gases en los
por litro de H2O] ∗10 6 = constante
líquidos, al contrario de la de los
P
175 0.000307 1.75 sólidos o líquidos, disminuye a medida
300 0.000500 1.67 que aumenta la temperatura. Un
414 0.000688 1.66 químico pondrá a hervir una muestra
610 0.00100 1.64 de agua para reducir la concentración
760 0.00128 1.68 de gases atmosféricos disueltos. Puesto
que la solubilidad del oxígeno en agua se reduce considerablemente con el calor, al vaciar
agua caliente en los ríos y lagos puede causar serio perjuicio a la vida acuática.

TABLA 3.4: Efecto de la temperatura sobre la


solubilidad• de los gases en agua••
Gas 0 ºC 10 ºC 50 ºC 100 ºC • La solubilidad se expresa en moles del gas disueltas por
H2 0.000960 0.000873 0.000717 0.000715 litro de agua
N2 0.00105 0.000830 0.000485 0.000423 •• La presión del gas encima de la solución es 1 atm.
O2 0.00212 0.00170 0.000933 0.000758
El efecto de la temperatura sobre la solubilidad
CO2 0.0765 0.0533 0.0194
de los gases se muestra en la información de la
tabla 3.4. La solubilidad del CO2 es mucho mayor que la de los otros tres gases, debido a que
el CO2 reacciona con el agua para forma ácido carbónico:

CO2 + H2O ⇒ H2CO3

3.5 EL PROCESO DE DISOLUCIÓN

U na sustancia puede disolverse con o sin reacción química en el disolvente. El sodio


metálico se disuelve en agua desprendiendo burbujas de hidrógeno y cantidades
considerables de calor, se produce un cambio químico en el cual el H2 y el hidróxido de sodio
iónico soluble, NaOH, son los productos, la ecuación iónica total será:

2Na(s) + 2H2O → 2[Na+(ac) + OH-(ac)] + H2(g) (1)

El cloruro de sodio sólido, por otra parte, se disuelve en agua sin evidencia de reacción
química:
NaCl(s) → Na+(ac) + Cl-(ac) (2)

Si la primera solución (1) se evapora a sequedad, se obtiene hidróxido de sodio sólido, NaOH,
en lugar de sodio sólido metálico. Esto junto con los productos de burbujas de hidrógeno,
indica que se efectúa una reacción con el disolvente. La evaporación de la solución de cloruro
de sodio (2) permite obtener en NaCl original. La facilidad del proceso de disolución depende
de dos factores: a) el cambio de energía (reacción exotérmica o endotérmica) y b) el cambio

COMPENDIOS DE QUÍMICA GENERAL PARA ESTUDIANTES DE CIENCIAS E INGENIERÍA


SOLUCIONES Y PROPIEDADES COLIGATIVAS 63
3
de desorden (entropía) que acompaña al proceso, es decir el proceso de disolución procede
hacia la disminución de la energía del sistema, lo cual corresponde a un proceso exotérmico y
hacia un incremento de desorden del sistema.

El primer factor que se refiere al cambio de energía se denomina calor de solución, ∆Hsolución.
En un líquido puro, las fuerzas intermoleculares se producen todas entre moléculas similares;
al mezclar un líquido con un sólido, cada molécula experimenta fuerza procedente de otras
moléculas o iones diferentes y también de moléculas similares. Las fuerzas relativas de estas
interacciones ayudan a determinar el grado de solubilidad de un soluto en un disolvente. Las
principales interacciones que afectan la disolución de un soluto en disolvente son:
1. Atracciones soluto-soluto

2. Atracciones disolvente-disolvente

3. Atracciones disolvente-soluto

La disolución se favorece cuando el valor de los dos primeros incisos son relativamente
pequeños y el del tercero es relativamente grande. Es preciso vencer las atracciones
intermoleculares o interiónicas entre las partículas de soluto para que se disuelva, esta parte
del proceso requiere consumo de energía. La separación de moléculas de disolvente también
consume energía. Sin embargo, cuando las partículas de soluto y las moléculas del disolvente
interaccionan en la solución se libera energía y el proceso de disolución es exotérmico.

Muchos sólidos se disuelven en líquidos por procesos 4endotérmicos. La razón de que estos
procesos se produzcan es que la endotermicidad es contrarrestada por un mayor incremento en
el desorden del soluto que acompaña al proceso de disolución. Casi todos los procesos de
disolución están acompañados de un incremento de desorden tanto en el disolvente como en
el soluto. Por tanto, este factor de desorden suele ser favorable a la solubilidad.

3.6 UNIDADES DE CONCENTRACIÓN

L a concentración de un soluto es la cantidad de soluto disuelto en una cantidad dada de


disolvente o de solución. La cantidad de disolvente o de solución se puede expresar en
términos de volumen o en términos de masa o de cantidad molar. Así, hay varias formas para
expresar la concentración de una solución.

La concentración de una solución suele expresarse como masa de soluto por unidad de
volumen; pero es mas significativa expresarla en términos de número de moles por unidad de
volumen. Los términos de concentración mas importantes utilizados por los químicos son:
molaridad, molalidad, normalidad y fracción molar.
3.6.1 Molaridad

3
Whitten Kennet W. define entropía como la medición del desorden de un sistema, mientras
mayor sea el desorden de un sistema, mayor será su entropía.
4
Proceso endotérmico es cuando en el proceso de disolución de un soluto, en el disolvente, éste absorbe energía
del medio ambiente.

COMPENDIOS DE QUÍMICA GENERAL PARA ESTUDIANTES DE CIENCIAS E INGENIERÍA


64 SOLUCIONES Y PROPIEDADES COLIGATIVAS

L a concentración molar , o molaridad [M], se define como los moles de soluto disueltos
en un litro de solución.
moles de soluto
Molaridad [ M ] = (3.1)
litro de solución

Una solución acuosa de HCl que es 0.2 M, contiene 0.2 moles de HCl por litro de solución, en
la práctica si se quiere preparar esta solución, se agrega a un matraz volumétrico aforado de
1.00  una cierta cantidad de agua por ejemplo ¼ , al cual añadimos 0.2 moles de HCl ( debe
hacerse cálculos a partir de los datos que lleva la etiqueta del frasco). En seguida se llena el
matraz con agua adicional a la marca hasta el cuello y se mezcla la solución.

Ejemplo 3.2.- 0.25 g de una muestra de sulfato de cobre pentahidratado (CuSO 4∗5H2O) se
coloca en un matraz aforado de 250 cc. El matraz se llena con agua hasta la marca en el
cuello. ¿Cuál es la molaridad de la solución resultante?

Solución.- Para determinar la molaridad, se necesitan los moles de soluto.


Por consiguiente convertiremos los gramos de CuSO4.5H2O a moles:

1 mol CuSO 4 ∗ 5H 2 O
n = 0.25 g CuSO 4 ∗ 5H 2 O ∗ = 0.1 mol CuSO 4 ∗ 5H 2 O
249.54 g

El volumen de la solución es 0.25 litros, en consecuencia la molaridad es:

0.1 mol CuSO 4 ∗ 5H 2 O


Molaridad = = 0.4 M
0.25 litros

3.6.2 Equivalente - gramo de ácidos y bases

S e define un equivalente – gramo de un ácido como el peso en gramos de éste que puede
producir un mol de H+ y un equivalente – gramo de una base como el peso en gramos de
ésta que pueda producir un mol de OH-. El peso equivalente o equivalente - gramo de un
ácido se obtiene dividiendo su peso fórmula o peso molecular entre el número de hidrógenos
ácidos que aporta una molécula del ácido. El peso equivalente o equivalente - gramo de una
base se obtiene dividiendo su peso fórmula o peso molecular entre el número de iones
hidróxido. A partir de definiciones anteriores, se ve que un equivalente gramo de cualquier
ácido reacciona con un equivalente – gramo de cualquier base.

No es cierto que un mol de cualquier ácido reacciona con un mol de cualquier base en una
reacción química específica. En consecuencia, según la definición de equivalentes, 1 eq-g
ácido = 1 eq-g base. Por lo general, se puede escribir la siguiente expresión para todas las
reacciones ácido base que llegan hasta el 5punto final.

Eq de ácido = eq de base o meq de ácido = meq de base (3.2)


Donde: meq = miliequivalentes, 1 eq = 1000 meq

3.6.3 Equivalente – gramo en reacciones Redox


5
Punto final es el punto en el cual el indicador cambia de color y se define la titulación en una reacción de
neutralización.
COMPENDIOS DE QUÍMICA GENERAL PARA ESTUDIANTES DE CIENCIAS E INGENIERÍA
SOLUCIONES Y PROPIEDADES COLIGATIVAS 65

E n reacciones de oxidación – reducción se debe recordar que un agente oxidante acepta


electrones, y un agente reductor los produce. El principio de equivalente permite hacer
cálculos estequiométricos en reacciones redox. El equivalente gramo del agente oxidante y del
agente reductor es respectivamente:
peso molecular
Eq − g (agente oxidante) =
No. e − ganados

peso molecular
Eq − g (agente reductor) =
No. e − perdidos
3.6.4 Normalidad

L a concentración normal , o normalidad [N], de una solución se define como el número


de pesos equivalentes o simplemente equivalentes (eq) de soluto por litro de solución.
Número de peso equivalente de soluto
Normalidad [ N ] = o
litro de solución
No. de miliequivalente de soluto
Normalidad[ N ] =
litro de solución

(3.3)

Ejemplo 3.3 ¿Cuál es la normalidad de una solución que contiene 10 g de H 2SO4 en 500 ml
de solución?

Solución.- El número de moles de H2SO4 es:

1 mol H 2SO 4
n = 10 g H 2SO 4 ∗ = 0.102 mol H 2SO 4
98 g H 2SO 4

El volumen de la solución es 0.5 litros, en consecuencia la normalidad es:

0.102 moles H 2 SO 4 2eq − gH 2 SO 4


Normalidad = ∗ = 0.408 N
0.5 litros 1molH 2 SO 4

3.6.5 Molalidad

L a concentración molal , o molalidad [m], se define como los moles de soluto disueltos
en un kilogramo de disolvente.
moles de soluto
Molalidad [ m] = (3.4)
kg de disolvente

COMPENDIOS DE QUÍMICA GENERAL PARA ESTUDIANTES DE CIENCIAS E INGENIERÍA


66 SOLUCIONES Y PROPIEDADES COLIGATIVAS
Ejemplo 3.4.- El tolueno, C6H5CH3, es un compuesto líquido similar al benceno, C6H6. Es la
materia prima para otras sustancias, entre ellas el trinitrotolueno (TNT). Encuentre la
molalidad del tolueno en una solución que contiene 5 g de tolueno en 25 g de benceno.

Solución.- El número de moles de tolueno es:


1 mol de C 6 H 5 CH 3
5 g C 6 H 5 CH 3 ∗ = 0.0543 mol C 6 H 5 CH 3
92 g C 6 H 5 CH 3

La masa del benceno que constituye el disolvente es: 0.025 kg C6H6

0.0543 mol C 6 H 5 CH 3 mol soluto


molalidad = = 2.172 = 2.172 molal
0.025 kg C 6 H 6 kg solvente

3.6.6 Formalidad

L a formalidad de una solución es el número de peso fórmula en gramos de soluto por litro
de solución; el símbolo de esta unidad es F. La formalidad es muy similar a la molaridad,
por ejemplo 1 formal que se simboliza 1F del ácido clorhídrico es igual a su peso fórmula, es
decir 36.5 g HCl, por litro de disolución.

3.6.6 Fracción molar

L a fracción molar de una sustancia A (χA), componente de una solución se define como
los moles de sustancia A divididos entre los moles totales de solución, esto es:
moles de sustancia A n
χA = = A (3.5)
moles totales de la solución n t
3.7 DILUCIÓN

E n la práctica de laboratorio con frecuencia se usan soluciones concentradas de


concentración conocida para preparar a partir de ellas soluciones diluidas. Por tanto, es
importante ver cómo los cambios de volumen afectan la concentración de una solución.

Cuando la concentración se expresa en una escala volumétrica, la cantidad de soluto


contenido en un volumen determinado de la solución es igual al producto del volumen por la
concentración, es decir:
Cantidad de soluto disuelto = volumen ∗ concentración

Cuando se diluye una solución, el volumen aumenta y la concentración disminuye, pero la


cantidad total de soluto permanece constante. Por esta razón, dos soluciones de
concentraciones diferentes pero que contengan las mismas cantidades de soluto, están
relacionadas por la siguiente expresión:

Volumen1 ∗ Concentración1 = Volumen2 ∗ Concentración2

V1 ∗ C1 = V2 ∗ C2 (3.6)

Para reacciones de neutralización ácido – base la ecuación correspondiente y de mucha


utilidad es:
COMPENDIOS DE QUÍMICA GENERAL PARA ESTUDIANTES DE CIENCIAS E INGENIERÍA
SOLUCIONES Y PROPIEDADES COLIGATIVAS 67
N1∗V1 = N2∗V2 (3.7)

Donde N1 y N2 son concentraciones normales y V1 y V2 son los volúmenes requeridos para la


neutralización de una base y una ácido. No obstante se usa también para diluciones.

Ejemplo 3.4.- 0.25  de una solución contiene 75 g de NaCl por litro. a) ¿A qué volumen se
debe diluir para obtener una solución de concentración 15 g de NaCl por litro?, b) ¿qué
cantidad de agua se necesita para este propósito?

Solución: a) Considerando la expresión (3.6) y despejando Volumen se tiene:

V1 ∗ C1 = V2 ∗ C2

V ∗C 0.25  ∗ 75 g/
V2 = 1 1 = = 1.25 
C2 15 g/

b) La cantidad de agua que se requiere para esta dilución es:

(1.25 – 0.25)  = 1 

Ejemplo 3.5.- Calcule el volumen aproximado del agua que se debe agregar a 250 cm 3 de una
solución 1.25 N para preparar una solución de concentración 0.5 N (desprecie los cambios en
volumen)

Solución: Considerando la expresión (3.7) se puede determinar inicialmente el volumen de la


nueva concentración y luego el volumen de agua requerido en la dilución de la concentración
original.
N1∗V1 = N2∗V2

N1V1 1.25 N ∗ 250 cm 3


V2 = = = 625 cm 3
N2 0.5 N
La cantidad de agua es:
(625 – 250) cm3 = 375 cm3

Ejemplo 3.6.- Si una solución se prepara a partir de 1 mol de etilenglicol y 9 mol de agua, los
moles totales de la solución son 10 moles, en consecuencia la fracción molar del etilenglicol
es:
1 mol etilenglicol
χ= = 0.1
10 moles

Recordemos que la suma de las fracciones de todos los componentes de una solución es igual
a 1.
n
(3.8) X =1∑
i =1
i
Algunas estrategias:
3.8 REACCIONES QUÍMICAS EN
SOLUCIONES La molaridad y la normalidad se calculan
partiendo de la densidad y del porcentaje

C omo es de conocimiento, se discutió el método de composición de una solución. En


soluciones diluidas la densidad de la
para resolver problemas en que intervienen solución es muy semejante a la densidad
reacciones químicas, ahora estamos preparados para del disolvente puro. Para calcular la
molalidad y fracción molar solo es
COMPENDIOS DE QUÍMICA GENERAL PARA ESTUDIANTES necesarioDE conocer
CIENCIAS el E INGENIERÍA
porcentaje de
composición de la solución.
68 SOLUCIONES Y PROPIEDADES COLIGATIVAS
extender el método a reacciones en solución y poder efectuar cálculos estequiométricos a
partir de un volumen de una concentración determinada a la cantidad de soluto presente.

Los problemas en esta reacción se basan en dos principios:

 El número de equivalentes de todas las especies en una reacción química es el mismo.


 Volumen ∗ normalidad = número de equivalentes

Ejemplo 3.7 ¿Cuántos mililitros de NaOH 4.0 N se necesitan para neutralizar 20 ml de HCl
3.0 N?

Solución: Se puede usar el concepto de miliequivalentes, esto es:


4.0 meq 3.0 meq
4.0 N = 3
y 3.0 N =
cm cm 3
Si: N1 = 4.0 N
N2 = 3.0 N
V2 = 20 ml HCl
V1 = ¿?
N 2 V2 3.0 ∗ 20 ml
V1 = = = 15 ml NaOH
N1 4.0

Ejemplo 3.8.- ¿Cuántos gramos de NaOH se necesitan para neutralizar 90 ml de HCl 1.5 M?

Solución: Para resolver el problema es conveniente transformar la concentración molar en


normalidad, esto es,
1 eq-g HCl = 1 mol HCl
En consecuencia:
moles HCl 1 eq − g HCl
1.5 ∗ = 1.5 N
 1 mol HCl

Por tanto, en una reacción ácido base se tiene:


No eq HCl = No eq NaOH
1.5 meq HCl
90 ml solución ∗ = 135 meq HCl = 135 meq NaOH
1 ml solución

Por consiguiente la masa de NaOH requerida es:


1 eq − g NaOH 40 g NaOH
135 meq NaOH ∗ ∗ = 5.4 g NaOH
1000 meq NaOH 1 eq − g NaOH

Ejemplo 3.9.-Encuentre el peso equivalente de KMnO4 en la siguiente reacción:

MnSO4 + KMnO4 + H2O ⇔ MnO2 + H2SO4 + K2SO4

b) ¿Cuántos gramos de MnSO4 se oxidan con 50 cm3 de una solución de KMnO4 0.1 N?

Solución: Para resolver el problema efectuaremos la igualación de la ecuación química.

[Mn+2 + SO4= ] + [K+ + MnO4- ] + H2O0 ⇒ MnO20 + [2H+ + SO4= ] + [2K+ + SO4=]

2∗ 3 e- + 4 H+ + MnO4- ⇒ MnO20 + 2 H2O0


COMPENDIOS DE QUÍMICA GENERAL PARA ESTUDIANTES DE CIENCIAS E INGENIERÍA
SOLUCIONES Y PROPIEDADES COLIGATIVAS 69

3∗ 2 H2O0 + Mn+2 ⇒ MnO20 + 4 H+ + 2 e-


2 MnO + 2 H2O + Mn+2
4
-
⇒ 5 MnO20 + 4 H+

La ecuación igualada es:

3 MnSO4 + 2 KMnO4 + 2 H2O ⇔ 5 MnO2 + 2 H2SO4 + K2SO4

a) el peso equivalente del permanganato de potasio es:


158 g
eq − g KMnO 4 = = 52.67 g
3
b) La masa de sulfato manganoso requerido es:
0.1 eq KMnO 4 1 mol KMnO 4 3 mol MnSO 4 151 g MnSO 4
50 cm 3 ∗ ∗ ∗ ∗ = 0.3775 g MnSO 4
1000 cm 3 3 eq KMnO 4 2 mol KMnO 4 1 mol MnSO 4

3.9 PROPIEDADES COLIGATIVAS

A lgunas propiedades físicas de las soluciones difieren mucho de las del disolvente puro.
Por ejemplo, el agua pura se congela a 0 ºC, pero las soluciones acuosas se congelan a
temperaturas menores. El etilenglicol se adiciona al agua de los radiadores de los automóviles,
pues es un anticongelante ya que abate (disminuye) el punto de congelación de la solución;
también eleva el punto de ebullición de la solución sobre la del agua pura, permitiendo que el
motor funcione a una temperatura mas alta.

Una solución, que conste de dos o mas componentes, carece de las propiedades físicas
constantes de una sustancia pura; estas propiedades dependen de la concentración de las
partículas del soluto y no de su naturaleza. Tales propiedades se conocen como propiedades
coligativas y éstas son: el descenso de la presión de vapor; la depresión en el punto de
congelación; la elevación en el punto de ebullición y la presión osmótica.

Las propiedades coligativas se pueden usar en la determinación de los pesos moleculares de


las sustancias disueltas y pueden dar además información acerca de las propiedades del soluto
si se conocen las propiedades del disolvente. Todas las soluciones obedecen las leyes que se
discuten en este capítulo , cuando la concentración es suficientemente baja. Por esta razón se
les conoce como leyes de las soluciones diluidas.

3.9.1 Descenso de la presión de vapor y la ley de Rault

M uchos experimentos han demostrado que las soluciones que contienen líquidos no
volátiles o sólidos como solutos, siempre tienen presiones de vapor mas bajas que los
disolventes puros.

Fig. A
FIGURA 3.6
La figura A muestra la presión de vapor del agua pura, la figura B el equilibrio de dos sistemas que contienen agua
pura, en la figua C la presión manométricaFig.(diferencia
B de altura) de los dos líquidos se debe al vapor de agua
Fig. C
pura. Esta presión es menor sobre la solución azucarada, por que hay menos moléculas de agua por unidad de área
de COMPENDIOS DE QUÍMICA GENERAL PARA ESTUDIANTES DE CIENCIAS E INGENIERÍA
superficie al evaporarse.
70 SOLUCIONES Y PROPIEDADES COLIGATIVAS

Cuando un soluto no volátil se disuelve en un líquido; parte del volumen total de la solución
es ocupada por moléculas de soluto, y por lo tanto, hay menos moléculas de disolvente por
unidad de área en la superficie. Esto ocasiona el descenso de presión de vapor del disolvente.
El descenso de la presión de vapor asociados con solutos no volátiles y no ionizables se
resume en la ley de Rault:
“La presión de vapor de un disolvente en una solución es igual a la presión de vapor del
disolvente puro multiplicada por la fracción molar del mismo en la disolución”.
Matemáticamente se expresa:
P = Xd.Pº (4.9)

Donde: Xd, representa la fracción molar del disolvente en la solución, Pº es la presión de


vapor del disolvente puro y P es la presión de vapor del disolvente en la solución. Esta ley
permite calcular pesos moleculares.

Ejemplo 3.10.- La presión de vapor del agua pura a 25 ºC es 23.76 mmHg y la presión de
vapor de una mezcla formada por 5.4 g de soluto no volátil en 90 g de agua es 23.32 mmHg.
Determinar el peso molecular de dicho soluto.

Solución: De acuerdo a la ley de Rault:


P = Xd.Pº

P 23.32
Xd = = = 0.9815
Po 23.76
Puesto que la fracción molar se define como:

moles de sustancia A n
XA = = A
moles totales de la solución n t

Se puede expresar en términos de sus pesos moleculares:


md
Md
0.9815 =
md m
+ s
Md Ms
reemplazando datos:
90 g
0.9815 = 18
90 g 5.4 g
+
18 Ms
despejando Ms:
Ms= 57.3 g/mol

Ejemplo 3.11.- Determine la presión de vapor a 25 ºC de una solución acuosa que consta de
10 g de sacarosa, C12H22O11 y 75 g de H2O.

Solución: De acuerdo a la ley de Rault:


COMPENDIOS DE QUÍMICA GENERAL PARA ESTUDIANTES DE CIENCIAS E INGENIERÍA
SOLUCIONES Y PROPIEDADES COLIGATIVAS 71
P = Xd.Pº
La fracción molar del disolvente es:
md
Md
Xd =
md m
+ s
Md Ms

4.17
χd = = 0.993
Por consiguiente La presión de 4.17 + 0.0292 vapor es: Pv = 0.993 ∗ 23.76
mmHg = 243.59 mmHg

3.9.2 La ley de Rault y las soluciones de solutos volátiles

E n soluciones en las cuales tanto el disolvente como el soluto tienen una presión de vapor
apreciable, se puede aplicar la ley de Rault a ambos componentes:

Para lograr cierta comprensión de tales mezclas, considere una solución ideal que contiene
dos componentes, A y B, según la ley de Rault:
PA = XA.PAº y PB = XB.PBº

La presión total del sistema es entonces, la suma de las presiones parciales de cada
componente volátil:
Ptotal = PA + PB = XA.PAº + XB.PBº (3.10)

Ejemplo 3.12.- ¿Cuál es la presión de vapor que ejerce una mezcla formada por 100 gramos
de benceno y 100 g de tolueno a 25 ºC? A dicha temperatura las presiones del vapor de
benceno y tolueno puros son, respectivamente 95.1 y 28.4 mmHg.

Solución: De acuerdo a la expresión (3.6)


Ptotal = PC6H6 + PC7H8

P total = XC6H6* PC6H6º + XC7H8 * PC7H8º


Las fracciones molares de las dos sustancias son:
100 g 100 g
n benceno = = 1.282 n tolueno = = 1.087
g g
78 92
mol mol
n total = 1.282 + 1.087 = 2.369
1.282 1.087
χ C6 H 6 = = 0.541 χ C7 H8 = = 0.459
2.369 2.369

La presión de vapor será:


Pt = 0.549* 95.1 mmHg + 0.459 * 28.4 mmHg

Pt = 52.21 + 13.04 = 65.25 mmHg

COMPENDIOS DE QUÍMICA GENERAL PARA ESTUDIANTES DE CIENCIAS E INGENIERÍA


72 SOLUCIONES Y PROPIEDADES COLIGATIVAS
Las soluciones pueden representarse gráficamente. La figura 3.7 muestra la ley de Rault para
una solución ideal de un soluto en
un líquido volátil. La presión de
vapor que ejerce el líquido es

PRESION DE VAPOR

PRESION DE VAPOR
DEL DISOLVENTE

DEL DISOLVENTE
proporcional a su fracción molar
en la solución.

Este diagrama se cumple para:


P = Xd.Pº
III
La figura 3.8 muestra la ley de
Rault aplicado a soluciones que II
tienen dos componentes volátiles.
Este diagrama se cumple para:
I
Ptotal = PA + PB = XA.PAº + XB.PBº

En la figura 3.8 la línea (I) es la A Xdisolvente B A Xdisolvente B


presión parcial de A y la línea (II) Figura 3.7 Figura 3.8
es la presión parcial de B y la
línea (III) es la presión total para diferentes concentraciones de los dos líquidos volátiles.

La figura 3.9 muestra una desviación negativa de la ley de Rault, Las fuerzas intermoleculares
existentes en la solución son superiores a las fuerzas intermoleculares de los componentes
puros aisladamente.

La figura 3.10 muestra la presión


de vapor superior a la presión
PRESION DE VAPOR
DEL DISOLVENTE

predicha por la ley de Rault


(desviación positiva). Las
fuerzas intermoleculares
existentes en la solución son más
débiles que las de los componentes
puros

A Xdisolvente B A Xdisolvente B
Figura 3.9 Figura 3.10
independientemente.

3.9.3 LEY DE HENRY

E l efecto de la presión sobre la solubilidad de un gas en un líquido se puede predecir de


manera cuantitativa. Para soluciones diluidas de un gas no reactivo en un líquido, puede
COMPENDIOS DE QUÍMICA GENERAL PARA ESTUDIANTES DE CIENCIAS E INGENIERÍA
SOLUCIONES Y PROPIEDADES COLIGATIVAS 73
aplicarse una expresión muy similar a la ley de Rault, conocida como la ley de Henry, cuya
expresión matemática es:
Pgas = K∗Xgas (3.11)

Donde Pgas es la presión del gas sobre la superficie de la solución y k es una constante para
un gas y un disolvente determinados a una determinada dada. Xgas representa la fracción
molar del gas disuelto; la relación es válida a bajas concentraciones y a bajas presiones.

Ejemplo 3.13.- Si 29 mg de N2 se disuelven en un litro de agua a 0 ºC y 760 mmHg de


presión de N2, ¿qué masa de N2 se disolverá en un litro de agua a 0 ºC y 5 atm de presión?

Solución: De acuerdo con la ecuación (3.11), en principio se determinará la constante de


Henry K, para luego determinar la masa de nitrógeno disuelto en las nuevas condiciones de
presión y concentración.

La fracción molar del gas es:


1g N2 1 mol N 2
29 mg N 2 ∗ ∗ = 1.036 ∗ 10 −3 mol N 2
1000 g N 2 28 g N 2

1000 ml 1 g 1 mol H 2 O
1 H 2 O ∗ ∗ ∗ = 55.56 mol H 2 O
1 H 2 O 1 ml 18 g H 2 O

1.036 ∗ 10 −3 molN 2
χ N2 = = 1.8646 ∗ 10 −5
55.561

La constante de Henry será:


Pgas 1 atm
K= = = 5.363 ∗10 4
χgas 1.8646 ∗10 −5

La masa de nitrógeno disuelta en 1 litro de agua a 5 atm será:


Pgas 5atm
χ N2 = = 4
= 9.323 ∗10 −5
K 5.363 ∗10
1 litro de H2O = 55.56 mol
nN 2 nN2
χ N2 = = = 9.323 ∗10 −5
n N 2 + n H 2O n N 2 + 55.56

Resolviendo la ecuación se tiene:


28 g N 2 1000 mg N 2
5.18 ∗ 10 −3 mol N 2 ∗ ∗ = 145.04 mg N 2
1 mol N 2 1 g N2

3.9.4 Aumento del punto de ebullición

R ecordemos que el punto de ebullición de un líquido es la temperatura a la cual la presión


de vapor se iguala a la presión aplicada en su superficie, por ejemplo la temperatura de
ebullición normal del agua sobre el nivel del mar es 100 ºC

COMPENDIOS DE QUÍMICA GENERAL PARA ESTUDIANTES DE CIENCIAS E INGENIERÍA


74 SOLUCIONES Y PROPIEDADES COLIGATIVAS
Se ha visto que la presión de vapor de un disolvente a una temperatura dada, desciende por la
presencia de un soluto no volátil. Las soluciones de este tipo deben calentarse a temperaturas
mas altas que el disolvente puro para que su presión de vapor iguale a la presión atmosférica.

El incremento en el punto de ebullición, ∆Te ( en relación al punto de ebullición del


disolvente puro), es directamente proporcional al número de partículas de soluto por mol de
disolvente. Sabemos que la molalidad expresa el número fijo de moles de disolvente. Así Te
PRESION DE VAPOR

es proporcional a la molalidad, como se muestra en la siguiente expresión matemática:


DEL DISOLVENTE

∆Te = Kem (3.12)

∆Te = Tf(solución) – Tf(disolvente)

Ejemplo 3.14.- Una disolución de glicocola preparada al disolver 1.62 g de sustancia en


19.62 g de agua, hierve a 100.6 ºC. Hallar el peso molecular de la glicocola. (Ke = 0.52
ºC/molal).

Solución: Datos:
Te = 100.6 ºC
Ke = 0.52 ºC/molal

De acuerdo a la relación (2.7) ∆Te = Kem

Se puede determinar la molalidad

ΔTe (100.6 − 100)°C


m= = = 1.154 molal
ke 0.52 °C/ molal

A partir de este dato se puede evaluar el número de moles de soluto:

moles de soluto
1.154 ∗ 19.62 g de agua = 0.0226 moles de soluto
1000 g de agua

En consecuencia el peso molecular de soluto será:

ms 1.62 g
M= = = 71.68 g/mol
n s 0.0226 moles

Para hallar el peso molecular se puede considerar un segundo método propuesto por el Prof.
José Ibarz Aznárez, el cuál expresa:

Si una disolución está constituida por a gramos de soluto y A g de disolvente, y el peso


molecular del soluto es M, la molalidad de disolución es:

a • 1000
m= (3.13)
A•M

Considerando la expresión (3.8), y despejando M se tiene:

COMPENDIOS DE QUÍMICA GENERAL PARA ESTUDIANTES DE CIENCIAS E INGENIERÍA


SOLUCIONES Y PROPIEDADES COLIGATIVAS 75
a • 1000 1.62 • 1000
M= = = 71.55 g/mol
A•m 19.62 • 1.154

3.9.5 Descenso del punto de congelación

E n contraste con el punto de ebullición, el punto de congelación de una solución


generalmente es mas bajo que el punto de congelación del solvente puro, como muestra
la figura 3.6. La diferencia entre estas dos temperaturas se conoce como depresión en el punto
de solidificación y se designa por ∆Tc, y es proporcional a la concentración molal del soluto.
Esta proporcionalidad, convertida en igualdad se puede expresar por medio de la siguiente
ecuación:

Tc = Kcm (3.14)

∆Tc = Tf(disolvente) – Tf(solución)

Presión de vapor [mmHg]


La figura 3.11 muestra como un soluto no
volátil abate la presión de vapor de un
disolvente, el punto de ebullición se eleva y
el punto de congelación desciende con
respecto a los puntos correspondientes en los
disolventes puros. La magnitud de elevación
del punto de ebullición ∆Te es menor que la
magnitud del abatimiento del punto de ∆P
congelación ∆Tc.

Ejemplo 3.15.- Se funde una mezcla de


0.436 g de acetanilida con 14.06 g de
alcanfor de punto de fusión 176.2 ºC. La
mezcla se deja solidificar y enfriar, se reduce ∆Tc Temperatura [ºC]
∆Te
a polvo, se mezcla íntimamente y se
Figura 3.11
calienta. En el momento en que se termina Diagrama de fase del H2O y de una solución acuosa
de fundir su temperatura es de 167.0 ºC.
Hallar el peso molecular de la acetanilida.
(kcalcanfor = 40.0 ºC/molal)

Solución: Los datos son los siguientes:


Tc = 176.2 ºC
kc = 40.0 ºC/molal

De acuerdo a la relación (3.7)


∆Tc = kcm

Se puede determinar la molalidad:

ΔTc (176.2 − 167.0)°C


m= = = 0.23 molal
kc 40.0 °C/ molal

COMPENDIOS DE QUÍMICA GENERAL PARA ESTUDIANTES DE CIENCIAS E INGENIERÍA


76 SOLUCIONES Y PROPIEDADES COLIGATIVAS

A partir de este dato se puede evaluar el número de moles de soluto:

moles de soluto
n = 0.23 ∗ 14.06 g de alcanfor
1000 g de alcanfor
n = 0.00323 moles de soluto

En consecuencia el peso molecular de soluto será:

ms 0.436 g
M= = = 134.98 g/mol
n s 0.00323 moles

Para hallar el peso molecular se puede considerar la expresión (3.8), y despejando M se tiene:

a • 1000 1.62 • 1000


M= = = 71.55 g/mol
A • m 19.62 • 1.154

TABLA 3.2
Algunas propiedades de disolventes comunes
PUNTO DE ke PUNTO kc
SOLVENTE EBULLICIO [ºC/m] FUSION [ºC/m]
N [ºC] [ºC]
H2O 100.00 0.52 0.0 1.86
C6H6 80.10 2.53 5.50 5.12
CCl4 76.80 5.02 -22.3 29.80
C2H5OH 78.40 1.22 -114.6 1.99
ClCHO 61.20 3.63 -63.50 4.68

3.9.6 Aplicaciones del aumento y descenso del punto de ebullición y fusión


respectivamente

Ejemplo 3.16.- El punto de ebullición de una solución de 0.402 g de naftaleno, en 26.6 g de


cloroformo, es 0.455 ºC mas alto que el del cloroformo puro. ¿Cuál es la constante
ebulloscópica del cloroformo?

Solución: A partir de la expresión (2.6) ∆Te = Kem, se puede despejar Ke, puesto que ∆Te
= 0.455° y la molalidad es:

0.402 g C10 H 8 1000 g CHCl3 1 mol C10 H 8


m= ∗ ∗ m = 0.118 molal
26.6 g CHCl3 1 kg CHCl 3 128 g C10 H 8
ΔTe 0.455 °C
Ke = = = 3.86 °C/molal
m 0.118 m

Ejemplo 3.17.- La presión de vapor de una solución acuosa diluida es 23.45 torr a 25 ºC,
mientras que la presión de vapor del agua pura a la misma temperatura es 23.76 torr.
Calcúlese la concentración molal del soluto, y utilícense los valores tabulados de Ke del agua
para predecir el punto de ebullición de la solución.

Solución: Puesto que la concentración molal se define como:

COMPENDIOS DE QUÍMICA GENERAL PARA ESTUDIANTES DE CIENCIAS E INGENIERÍA


SOLUCIONES Y PROPIEDADES COLIGATIVAS 77
moles de soluto
Molalidad [ m] =
kg de disolvente

se puede considerar la masa del disolvente como 1 kg = 1000 g H2O.

De acuerdo a la ley de Rault:


P = Xd.Pº

P 23.45
Xd = o
= = 0.987
P 23.76

Puesto que la fracción molar se define como:


moles de sustancia A n
XA = = A
moles totales de la solución n t

Se puede expresar en términos de sus pesos moleculares:


md
Md
0.987 =
md m
+ s
Md Ms

reemplazando datos:
1000 g
0.987 = 18
1000 g
+ ns
18

El número de moles del soluto, despejando de la anterior expresión es en consecuencia:


ns = 0.727 mol
y la molalidad será:
0.727 mol de soluto
m= = 0.727 molal
1 kg de agua
El punto de ebullición de la solución es:

De acuerdo a la relación (3.7)


∆Te = Kem

∆Te = 0.52 °C/m ∗ 0.727 m = 0.38 °C

La temperatura de ebullición de la solución es:


Te = (100 + 0.38) °C = 100.38 °C

3.10 OSMOSIS Y PRESIÓN OSMÓTICA

L a osmosis es el proceso espontáneo por el cual las moléculas del disolvente atraviesan
una membrana semipermeable de una solución de menor concentración de soluto hacia
una solución con mayor concentración de mayor soluto

COMPENDIOS DE QUÍMICA GENERAL PARA ESTUDIANTES DE CIENCIAS E INGENIERÍA


78 SOLUCIONES Y PROPIEDADES COLIGATIVAS
Para definir la presión osmótica es conveniente considerar el sistema de la figura 2.12. El cual
muestra un experimento en una cámara de presión osmótica.

Rebalse de la solución Figura 3.14


Solución de azúcar Las moléculas de
Agua Figura 3.13 azúcar no pueden
atravesar la
membrana
H = π = ρgh

Membrana Las moléculas de agua


Figura 3.12 atraviesan la membrana Disolvente
puro
(Agua)
Figura 3.12 - 3.14
Experimento de ósmosis. El agua pasa a través de la membrana a la solución de azúcar en el
compartimiento del sistema. El flujo de agua cesa cuando el líquido en el embudo ejerce hacia abajo una
presión suficiente (la presión osmótica).

a) La figura 3.12 muestra el inicio de la determinación de la presión osmótica, los niveles de


solución del lado izquierdo y del disolvente del lado derecho son iguales.

b) Después del inicio del experimento, las moléculas del disolvente tienden a fluir hacia la
solución, entonces observamos rebalse de la solución como era de esperarse, como muestra la
figura 3.13.

c) Para evitar el rebalse instalemos un tubo en la cámara de la disolución; durante cierto


tiempo de iniciado el experimento, el flujo de la moléculas hacia la solución cesa y el sistema
alcanza el equilibrio cuando el menisco se ha elevado hasta una determinada altura, como se
Fig. 2.13
puede observar en la figura 3.14. En estas condiciones de equilibrio, la solución se encuentra
bajo una presión hidrostática mayor que el disolvente puro. La altura del menisco
multiplicada por la densidad de la solución y la aceleración de la gravedad, dá la presión
adicional sobre la solución y ésta es la presión osmótica π.
Por la medición experimental realizada en soluciones diluidas de concentración conocida, se
sabe que la relación entre la presión osmótica y la concentración está dada simplemente por:

π = cRT (3.15)

Donde c es la concentración de la disolución en moles/litro, R es la constante universal de los


gases y T es la temperatura absoluta. La presión osmótica es proporcional a la temperatura
absoluta porque afecta el número de colisiones del disolvente con la membrana por unidad de
tiempo, también es proporcional a la concentración molar, ya que ésta afecta a la diferencia en
el número de moléculas del disolvente que chocan contra la membrana de cada lado.

Ejemplo 3.18.- Estime el peso molecular de una macromolécula biológica si se disuelve una
muestra de 0.194 gramos en 82.4 ml de benceno y se observa una presión osmótica de 14.20
torr a 25 °C.

Solución: De la ecuación (3.15) π = cRT:


π 14.20 mmHg −4 mol
c = GENERAL
COMPENDIOS DE QUÍMICA = PARA = 7.64 ∗ 10
mmHg −  ESTUDIANTES DE CIENCIAS E INGENIERÍA
RT 
62.4 ∗ 298K
K − mol
SOLUCIONES Y PROPIEDADES COLIGATIVAS 79

Considerando que el volumen de la solución es 82.4 ml, se puede determinar el número de


moles de la macromolécula biológica:
mol
7 ∗ 10 − 4 ∗ 0.0824 = 5.768 ∗ 10 − 5 mol

El peso molecular de dicha sustancia biológica es:
m 0.194g g
M= = −5
= 3363.38
n 5.768x10 mol
3.10.1 Ósmosis Inversa

E l6 proceso de ósmosis inversa se ha aplicado al problema de la purificación del agua, en


particular el método se ha utilizado para desalar el agua del océano (esto es para eliminar
lar sales del agua de mar y obtener agua que se pueda beber o que sea industrialmente
utilizable). En la ósmosis normal, el disolvente fluye a través de una membrana de una
solución diluida a una solución mas concentrada, el proceso de ósmosis se puede detener.
Figura 3.15
Sistema de desalación que utiliza la ósmosis inversa.
Cuando el agua del océano se somete a una presión mayor que
su presión osmótica, el agua pura pasa a través de una serie de
membranas y dejan detrás una solución de sal más
concentrada. Si se aplica una presión aún mayor, el proceso
osmótico puede ser invertido, entonces el disolvente fluye de
la solución concentrada (que pueda ser agua del océano), a
través de una membrana, a una solución más diluida (que
pueda ser agua mas o menos pura). En la figura 3.15 se
presenta un sistema que utiliza ósmosis inversa para desalar el
agua del océano.

3.11 Propiedades Coligativas de las


Soluciones Ionicas

P ara explicar las propiedades coligativas de las soluciones iónicas, debe aceptarse que la
concentración total de iones, es mas importante que la concentración de una sustancia
iónica. Por ejemplo, la depresión de congelación de una solución 0.100 m de cloruro de sodio
es casi el doble del de una solución 0.100 m de glucosa. Puede explicarse esto diciendo que el
cloruro de sodio se disuelve en agua para formar los iones Na+ y Cl-. Cada unidad fórmula de
NaCl da dos partículas.

Para cada propiedad coligativa de las soluciones iónicas se pueden considerar tres valores: el
valor calculado ∆Tm, suponiendo el compuesto formado por moléculas; el valor real, ∆T,
bastante mayor, encontrado experimentalmente; y el valor ideal ∆Ti, mayor todavía, que
puede también calcularse al suponer el compuesto formado por iones que se comportasen en
la disolución como si fueran partículas neutras..

3.11.1 Factor de van’t Hoff

La relación:
ΔT
i= (3.16)
ΔTm

COMPENDIOS DE QUÍMICA GENERAL PARA ESTUDIANTES DE CIENCIAS E INGENIERÍA


80 SOLUCIONES Y PROPIEDADES COLIGATIVAS

donde i es mayor a la unidad, se conoce como factor de van’Hoff. Cuando se suponía que los
electrolitos estaban constituidos por moléculas se aceptó que una fracción de las mismas se
disociaba en iones y que se establecía un equilibrio entre las moléculas sin disociar y los iones
formados. La fracción de las moléculas ionizadas o disociadas se denomina grado de
disociación. Para los electrolitos fuertes se establece actualmente la relación:

ΔT
=g (3.17)
ΔTi

donde g es menor que la unidad y que se conoce como coeficiente osmótico. Cuanto mas se
acerca a la unidad el valor de g mayor es el comportamiento ideal de los iones en la
disolución iónica.

Si una supuesta molécula del electrolito se disocia en ν iones, es evidente que:

∆Ti = ν∆Tm , y por tanto;

i
g= (3.18)
υ

En el caso de los electrolitos débiles, si α es el grado de ionización, y una molécula forma


realmente ν iones, 1 mol del electrolito dará lugar a να moles y quedaran sin ionizar ( 1 - α)
moles, por lo cual, en vez de un mol de compuesto tendremos:

[1+(ν - 1)α]

moles de partículas, y puesto que cualquier propiedad coligativa es i veces mayor que el valor
teórico correspondiente al numero de moléculas o moles disueltos, tendremos

i = 1 + (ν - 1)α (3.19)
y
i −1
α= (3.20)
υ −1

Esta expresión se aplica también corrientemente a los electrolitos fuertes, aunque ya se ha


indicado que es incorrecto hablar en ellos de grado de disociación. Para no romper con esta
costumbre nos referiremos nosotros a un grado de disociación aparente para explicar el
comportamiento de los electrolitos fuertes.

El factor de van’t Hoff i para soluciones iónicas en las expresiones de las propiedades
coligativas se puede escribir:

Descenso de la presión de vapor


md
P = P° Md
md ms
(3.21)
+i
Ms Ms
Aumento del punto de ebullición
∆Te = iKem (3.22)
Descenso del punto de fusión
COMPENDIOS DE QUÍMICA GENERAL PARA ESTUDIANTES DE CIENCIAS E INGENIERÍA
SOLUCIONES Y PROPIEDADES COLIGATIVAS 81
∆Tc = iKcm (3.23)
Presión osmótica
π = icRT (3.24)

Ejemplo 3.19.- A 100 °C la presión de vapor de una disolución de 10.0 g de nitrato cálcico en
150 g de agua es de 746.8 mmHg. Hallar el grado de disociación aparente del nitrato cálcico
en esta disolución.

Solución: Para hallar el grado de disociación aparente (véase ecuación 3.15) del nitrato
cálcico (electrolito fuerte) debemos determinar el coeficiente de van’t Hoff a partir de la
ecuación (3.16) y el número de iones ν

Ca(NO3)2 = [ Ca++ + 2NO3-] ν=3


Así que, de la ecuación (3.16):
md
P = P° Md
md ms
+i
Ms Ms

reemplazando datos:
150
746.8 = 760 18
150 10
+i
18 164

efectuando operaciones con una máquina electrónica:

8.33
0.982 =
8.33 + 0.0608 i

i = 2.42
Aplicando la expresión (3.15)
i − 1 2.42 − 1
α= = = 0.71
υ −1 3−1

Ejemplo 3.20.- Una disolución 0.065 molal de cloruro cálcico empieza a congelar a – 0.322
°C, hallar: a) el grado de disociación del cloruro cálcico y b) la concentración de los iones
cloruro y de los iones calcio en la disolución. Suponer la densidad de la disolución igual a la
unidad. Kc(H2O = 1.86 °C/mol)

Solución: a) Para hallar el grado de disociación haremos uso de la ecuación (2.15), sin
embargo es preciso evaluar i y ν.

El cloruro de calcio se disocia:


CaCl2 = [ Ca++ + 2Cl- ] ν=3

De la ecuación (3.18) ∆Tc = iKcm: Despejamos i:


ΔTc [ 0 − (−0.322) ]
i= = = 2.66
Kc ∗ m 1.86 ∗ 0.065

COMPENDIOS DE QUÍMICA GENERAL PARA ESTUDIANTES DE CIENCIAS E INGENIERÍA


82 SOLUCIONES Y PROPIEDADES COLIGATIVAS
Por consiguiente
2.66 − 1
α= ∗ 100 = 0.83
3 −1

b) Para hallar la concentración molar de los iones, determinaremos en primera instancia la


concentración del cloruro de calcio: La masa total de la solución se puede calcular:

0.065 mol CaCl 2 111 g CaCl 2 7.215 g CaCl 2


∗ =
1000 g H 2 O 1 mol CaCl 2 1000 g H 2 O

Es decir la masa de la solución es:


msolución = 1000 g
puesto que la densidad es 1.0 g/ml

El volumen de la solución es:


V = 1000 ml = 1.000 

La concentración molar de la solución iónica de CaCl2 es:

0.065 mol de CaCl 2


M= = 0.065 molar
1.000 
y las concentraciones de los iones determinamos a partir de : 0.065 ∗ 0.83 = 0.0540
CaCl2 = Ca++ + 2Cl-
1 mol 1 mol 2 mol

[ Ca++ ]= 0.0540 molar, la relación es 1:1

[ Cl- ] = 2 ( 0.0540) = 0.108 molar la relación es 1:2


PROBLEMAS RESUELTOS

3.1 Una solución de ácido sulfúrico tiene densidad de 1.84 g/ml y contiene 98% en masa de
H2SO4. ¿Qué volumen ocuparán 200 g de H2SO4 puro?

Solución: Se puede considerar los conceptos básicos de factores de conversión:


100 g de solución 1 cm 3 de solución
200 g H 2SO 4 ∗ ∗ = 110.91 cm 3 de solución
98 g H 2SO 4 1.84 g de solución

110.91cm 3 = VH 2 O + VH 2 SO 4 (1)
El volumen de agua es:
100 g de solución 2 g de H 2 O 1 cm 3 H 2 O
200 g H 2SO 4 ∗ ∗ ∗ = 4.082 cm 3 H 2 O
98 g H 2SO 4 100 g de solución 1 g H 2 O

En consecuencia, considerando la ecuación (1), el volumen de H2SO4 puro es:

VH 2SO 4 = 110.91 − 4.082 = 106.82cm 3 H 2SO 4

COMPENDIOS DE QUÍMICA GENERAL PARA ESTUDIANTES DE CIENCIAS E INGENIERÍA


SOLUCIONES Y PROPIEDADES COLIGATIVAS 83

3.2 Calcule la molalidad, fracciones molares de soluto y disolvente, y la molaridad de las


siguientes soluciones: a) una solución acuosa de H2SO4 que es 50 % en masa y tiene una
densidad de 1.4 g/ml, b) una solución acuosa de sacarosa, C12H22O11, que es 19 por ciento
sacarosa en masa, y tiene una densidad de 1.08 g/ml, y c) una solución compuesta de 24.4 g
de NaOH y 97.6 de H2O con un volumen de 100 ml.

Solución: a) La molalidad de la solución de H2SO4 se puede determinar a partir de la


composición centesimal, vale decir:

50 g H 2SO 4 1000 g H 2 O 1 mol H 2SO 4


m= ∗ ∗ = m = 10.2 molal
50 g H 2O 1 kg H 2 O 98 g H 2SO 4

La fracción molar se determina calculando previamente los moles del soluto y disolvente:

1 mol H 2SO 4
50 g H 2SO 4 ∗ = 0.510 mol H 2SO 4
98 g H 2SO 4
1 mol H 2 O
50 g H 2 O ∗ = 2.78 mol H 2O
18 g H 2 O
La fracción molar será entonces :

0.510
X H 2 SO 4 = = 0.155
3.290
X H2O = 1 − 0.155 = 0.845

La molaridad se calcula considerando la densidad de la solución

g Solución 50 g H 2SO 4 1 mol H 2SO 4 1000 mlSol.


M = 1.4 ∗ ∗ ∗ = 7.14 molar
ml Solución 100 g Solución 98 g H 2SO 4 1 Solución

b) La molalidad de la solución de C12H22O11, se puede determinar a partir de la composición


centesimal, vale decir:

19 g C12 H 22 O11 1000 g H 2O 1 mol C12 H 22 O11


m= ∗ ∗ = 0.6859 molal
81 g H 2 O 1 kg H 2 O 342 g C12 H 22 O11

La fracción molar se determina calculando previamente los moles del soluto y disolvente:

1 mol C12 H 22 O11


n C12 H 22 O11 = 19 g C12 H 22O11 ∗ = n C12 H 22 O11 = 0.0555 mol C12 H 22 O11
342 g C12 H 22 O11
1 mol H 2O
81 g H 2 O ∗ = 4.50 mol H 2 O
18 g H 2 O

La fracción molar será entonces :


0.0555
X C12 H 22 O11 = = 0.012
4.5555
X H2O = 1 − 0.012 = 0.988

COMPENDIOS DE QUÍMICA GENERAL PARA ESTUDIANTES DE CIENCIAS E INGENIERÍA


84 SOLUCIONES Y PROPIEDADES COLIGATIVAS
La molaridad se calcula considerando la densidad de la solución

g Solución 19gC12 H 22 O11 1 mol C12 H 22 O11


M =1.08 ∗ ∗ ∗
ml Solución 100 g Solución 342 g C12 H 22 O11
1000 ml Solución
= 0.6 molar
1 Solución

3.3 Una solución acuosa de cloruro de sodio que contiene 8.00 g de NaCl por 100 g de
solución posee una densidad de 1.054 g/ml a la temperatura de 25ºC. Determine: a) la
concentración molar y b) la concentración molal de la solución.
Solución:
Nº moles de soluto
a) 8 g NaCl/100 g sol. Molaridad =
1 litro de solución
ρ = 1.054 g/ml
moles de soluto :
m 8g
n= = = 0.137 mol NaCl
M 58.5 g/mol

-Cálculo del volumen en litros de solución:


1 ml de solución
V = 100 g ∗ = 94.88 ml
1.054 g

V= 0.0949 litros
Molaridad :
0.137 mol NaCl
M= = 1.44 molar
0.0949 
Nº de moles de soluto
b) Molalidad =
kg de solvente

Cálculo de la masa en kilogramos de solvente


Masa del solvente = masa (solución) - masa (soluto) = 100 g - 8 g = 92 g de solvente
Kg de solvente = 92g = 0.092 Kg
0.137 mol de soluto
Molalidad = = 1.49 molal
0.092 kg de solvente

3.4 Se prepara 1.5 litros de una solución de H2O y C2H5OH, cuya densidad relativa resulta
0.947 en una proporción volumétrica de 3:1 respectivamente. Determinar: a) La densidad del
alcohol etílico. b) La fracción molar del alcohol etílico. c) La molaridad. d) La molalidad.

Solución: Datos:
Vsolución = 1.5 
ρsolución = 0.947 kg/
%VH2O = 75%
%VC2H5OH = 25%

a) VH2O = 1.5  ∗ 0.75 = 1.125 

COMPENDIOS DE QUÍMICA GENERAL PARA ESTUDIANTES DE CIENCIAS E INGENIERÍA


SOLUCIONES Y PROPIEDADES COLIGATIVAS 85
VC2H5OH = 1.5  ∗ 0.25 = 0.375 
kg
m sol m sol = ρ sol ∗ Vsol = 0.947 ∗ 1.5
ρ sol = 
Vsol
m sol = 1.4205 kg

En consecuencia la masa del alcohol etílico es:


mC2H5OH = msol – mH2O

mC2H5OH =1.4305 – 1.125 = 0.2955 kg

mC2H5OH = 295.5 g
La densidad del alcohol etílico es:
m 295.5g
ρ= = = 0.788g/ml
V 375ml
n C H OH
b) χ C 2 H 5 OH = 2 5
nT

295.5 g
n C 2 H 5 OH = = 6.424 mol
g
46
mol

1125 g
n H 2O = = 62.5 mol
g
18
mol

nT = 6.424+62.5= 68.924 mol

6.424mol
χ C2 H 5OH = = 0.093
68.924mol

c)
n soluto 6.424 C 2 H 5 OHmol
M= = = 4.28 molar
 solución 1.5 solución

d) n soluto 6.424 molC 2 H 5 OH


m= = = 5.71 molal
kg dilsolvente 1.125 kg H 2 O

3.5 ( 20 puntos, 20 min) La densidad relativa de una disolución acuosa de cloruro de potasio
que contiene 24.6 g KCl es de 1.131 a 21 ºC, mientras que la densidad relativa del cloruro de
potasio sólido, a la misma temperatura es 1.984. Calcular: a) la molaridad, b) la normalidad,
c) la molalidad, d) la fracción molar, y e) el tanto por ciento en volumen. (K = 39; Cl = 35.5)

Solución: Datos: ρSolución= 1.131 g/ml; ρKCl= 1.984 g/ml; ρH2O= 1.0 g/ml;
mKCl = 24.6 g mH2O = x
m m KCl + m H 2O
ρ solución = total = (1)
Vtotal V KCl + V H 2O
mH2O = VH2O (2)
m 24.6 g
V KCl = KCl = = 12.40ml (3)
ρ KCl 1.984
Reemplazando en (1)
COMPENDIOS DE QUÍMICA GENERAL PARA ESTUDIANTES DE CIENCIAS E INGENIERÍA
86 SOLUCIONES Y PROPIEDADES COLIGATIVAS
24.6 g + m H 2O
1.131 =
12.4 + m H 2O
mH2O = 80.76 g

VH2O = 80.76 ml
Volumen total:
Vsolución = 12.4 ml + 80.76 ml = 93.16 ml

a) la molaridad es:
24.6 gKCl 1000ml 1molKCl
∗ ∗ = 3.54molar
93.16ml 1 74.5 gKCl
b) La normalidad es:

24.6 gKCl 1000ml 1molKCl 1eq − gKCl


∗ ∗ ∗ = 3.54normal
93.16ml 1 74.5 gKCl 1molKCl
c) La molalidad:

24.6 gKCl 1000 gH 2O 1molKCl


∗ ∗ = 4.09molal
80.76 gH 2O 1KgH 2O 74.5 gKCl
d) La fracción molar:

1molKCl 1molH 2 O
24.6 gKCl ∗ = 0.33molKCl 80.76 gH 2 O ∗ = 4.49molH 2 O
74.5 gKCl 18 gH 2 O
0.33 4.49
X KCl = = 0.068 X H 2O = = 0.932
4.82 4.82
12.4ml
e) El % en volumen: %V KCl = ∗ 100% = 13.31%
93.16ml

3.6.- Determinar: a) la concentración molal, b) la fracción molar y la molaridad


de una solución de ácido sulfúrico del 50% en masa y cuya densidad es 1.4 g/ml.

Solución: a) La molalidad de la solución de H2SO4 se puede determinar a partir de la


composición centesimal, vale decir:

50 g H 2SO 4 1000 g H 2O 1 mol H 2SO 4


m= ∗ ∗ = 10.2 molal
50 g H 2 O 1 kg H 2 O 98 g H 2SO 4

La fracción molar se determina calculando previamente los moles del soluto y disolvente:

1 mol H 2SO 4
50 g H 2SO 4 ∗ = 0.510 mol H 2SO 4
98 g H 2SO 4
1 mol H 2 O
50 g H 2 O ∗ = 2.78 mol H 2 O
18 g H 2 O
La fracción molar será entonces :
0.510
X H 2SO 4 = = 0.155
3.290
X H2O = 1 − 0.155 = 0.845
La molaridad se calcula considerando la densidad de la solución
g Solución 50 g H 2SO 4 1 mol H 2SO 4 1000 ml Solución
M = 1.4 ∗ ∗ ∗ = 7.14 molar
COMPENDIOSmlDESolución
QUÍMICA GENERAL
100 g SoluciónPARA98 gESTUDIANTES
H 2SO 4 DE CIENCIAS E INGENIERÍA
1 Solución
SOLUCIONES Y PROPIEDADES COLIGATIVAS 87

3.7 A 100 ml de una disolución de ácido sulfúrico del 96% en masa y de densidad relativa
1.84, se añadieron 400 ml de agua, obteniéndose una solución de peso específico relativo
igual a 1.22. En la solución resultante determinar:
a) la concentración en tanto por ciento en masa.
b) La concentración molar
c) La concentración molal
d) La normalidad
e) La fracción molar

Solución: El lector debe hacer énfasis en esquemas de manera que le permitan ver
objetivamente el problema:
H2O
V = 100 ml
V = ¿?
96%H2SO4
%H2SO4 =¿?
ρ = 1.84
ρ = 1.22

A 100 ml de una solución de ácido sulfúrico se añade 400 ml de agua, a esta operación se
denomina dilución, que obviamente modificará la concentración y la densidad de la solución.

Para resolver el problema calcularemos el volumen de agua de la solución original para luego
sumar a los 400 ml de agua que se añadieron, a partir de este dato la masa de agua de modo
que sumados a la masa de ácido sulfúrico puro y considerando la densidad de la solución
resultante se podrá determinar el volumen de la solución resultante.

Determinación de la masa de H2O y H2SO4 de la solución original

1.84 g de sol.
100 ml de sol. ∗ = 184 g de solución
1 ml de sol.
96 g H 2SO 4
184 g de solución ∗ = 176.64 g H 2SO 4 la masa de agua en la
100g de solución
solución resultante
m agua = 184 g − 176.64 g = 7.36 g H 2 O
será:

mH2O = (7.36 + 400) g = 407.36 g H2O

De acuerdo a algunas estrategias recomendadas en el presente texto es fundamental conocer la


concentración de la solución resultante en tanto por ciento:

a) El tanto por ciento del ácido sulfúrico se determina a partir de:


m H2SO4 = 176.64 g
m H2O = 407.36 g

La masa total es: (176.64 + 407.36) g = 584 g

176.64
%H 2SO 4 = ∗ 100% = 43.36%
407.36

COMPENDIOS DE QUÍMICA GENERAL PARA ESTUDIANTES DE CIENCIAS E INGENIERÍA


88 SOLUCIONES Y PROPIEDADES COLIGATIVAS
b) La concentración molar se determina conociendo la densidad de la solución y la
concentración en tanto por ciento en masa de ácido sulfúrico:

g solución 1000 ml solución 43.36 g H 2SO 4 1 mol H 2SO 4


M = 1.22 ∗ ∗ ∗
ml solución 1 de solución 100 g solución 98 g H 2SO 4

M = 5.398 molar
c) Determinación de la molalidad:

Sólo se considera la concentración en tanto por ciento en masa:


Masa de H2SO4 = 43.36 g
Masa de H2O = (100 - 43.36) g = 56.64 g

En consecuencia:
43.36 g H 2SO 4 1000 g H 2 O 1 mol H 2SO 4
molalidad = ∗ ∗ = 7.811 molal
56.64 g H 2 O 1 kg H 2 O 98 g H 2SO 4

d) Determinación de la normalidad:

Se puede calcular a partir de la molaridad considerando que en esta solución:

1 mol H 2SO 4
1 eq − g H 2SO 4 =
2
2 eq − g H 2SO 4 = 1 mol H 2SO 4

Por tanto:
moles H 2SO 4 2 eq − g H 2SO 4
5.398 ∗ = 10.796 N
1 solución 1 mol H 2SO 4
d) Determinación de la fracción molar:

Para calcular la fracción molar requerimos los moles del soluto y del disolvente:

1 mol H 2SO 4
nH 2SO 4 = 176.64 g H 2SO 4 ∗ = 1.802 mol
98 g H 2SO 4
1 mol H 2 O
nH 2O = 407.36 g H 2 O ∗ = 22.63 mol
18 g H 2 O
Las fracciones molares son:
1.80
χ H 2 SO 4 = = 0.07
24.43
22.63
χ H 2O= = 0.93
24.43
3.8 Una instalación de cámaras de plomo suministra diariamente 12 m3 de ácido de cámara de
densidad relativa 1.49 y de una concentración del 60% en masa. Si se desea preparar un ácido
concentrado del 98%

a) ¿Qué cantidad de agua se debe evaporar diariamente?

b) Calcular la normalidad, molaridad y la molalidad del ácido de cámara.

COMPENDIOS DE QUÍMICA GENERAL PARA ESTUDIANTES DE CIENCIAS E INGENIERÍA


SOLUCIONES Y PROPIEDADES COLIGATIVAS 89

Solución: El ácido de cámara del que se habla no es más que el ácido sulfúrico.

a) Efectuaremos un balance másico sin reacción química puesto que se trata de un proceso
físico, donde la masa de la solución mA de ácido de cámara es:

t
m A = 12 m 3 ∗ 1.49 = 17.88 t
m3
7
Asumiremos como base de cálculo 17.88 t por día.
Ácido de cámara A
(60%) H2SO4

B C

Agua evaporada Ácido resultante


(0%) H2SO4 ( 98%) H2SO4

mA = mB + mC (1)

amA = bmB + cmC (2)

En la ecuación (2) b = 0, por tanto la masa la masa del ácido resultante mc es:

am a 60 ∗ 17.88 t
mc = = = 10.947 t
c 98

De la ecuación (1) despejamos la masa de agua:

mB = mA - mC

mB = 17.880 - 10.947 = 6.933 t H2O

Puesto que la densidad del agua es 1 t/m3

El volumen de agua que se evapora diariamente es de 6.933 m3

b) La concentración molar del ácido de cámara se puede determinar a partir de su densidad y


tanto por ciento en peso.

g solución 1000 ml solución 60.00 g H 2SO 4 1 mol H 2SO 4


M = 1.49 ∗ ∗ ∗ = 9.12 molar
ml solución 1 de solución 100 g solución 98 g H 2SO 4

La normalidad se puede calcular a partir de la molaridad considerando que en esta solución:

7
5 Fuente: Viceministerio de Industria y comercio interno. De acuerdo al Sistema Internacional de unidades el
símbolo de tonelada métrica es t

COMPENDIOS DE QUÍMICA GENERAL PARA ESTUDIANTES DE CIENCIAS E INGENIERÍA


90 SOLUCIONES Y PROPIEDADES COLIGATIVAS
1 mol H 2SO 4
1 eq − g H 2SO 4 =
2
2 eq − g H 2SO 4 = 1 mol H 2SO 4
Por tanto:
moles H 2SO 4 2 eq − g H 2SO 4
9.12 ∗ = 18.24 N
1 solución 1 mol H 2SO 4

Para determinar la molalidad sólo se considera la concentración en tanto por ciento en masa:

masa de H2SO4 = 60.0 g


masa de H2O = (100 – 60.0) g = 40.0 g

En consecuencia:
60.0 g H 2SO 4 1000 g H 2 O 1 mol H 2SO 4
molalidad = ∗ ∗ = 15.31 molal
40.0 g H 2 O 1 kg H 2 O 98 g H 2SO 4

3.9 Una disolución de hidróxido ferroso a 25 ºC de temperatura tiene una solubilidad de 1.78
g/100g H2O. Determinar: a) la molaridad, b) la normalidad, c) la molalidad y la fracción
molar del soluto suponiendo solución ideal.

Solución: a) Cuando la solución es ideal la densidad de la solución tiende a ser la densidad del
agua, sin embargo la masa total de la solución se debe considerar:

(100 g + 1.78g) = 101.78 g

1.78 g Fe(OH)2 1 mol Fe(OH)2 1 g solución 1 000 cm 3solución


∗ ∗ ∗ = 0.194 molar
101.78 g solución 90 g Fe(OH)2 1 cm3 solución 1 litro de solución

b) Para determinar la normalidad de la solución, hallamos la relación de moles y equivalentes


gramo del soluto:
1 mol Fe(OH)2
1 eq − g Fe(OH)2 =
2
2 eq − g Fe(OH)2 = 1 mol Fe(OH)2

mol (FeOH)2 2 eq − g Fe(OH)2


0.194 ∗ = 0.388 N
1 de solución 1 mol Fe(OH)2

c) A partir de la solubilidad es posible calcular la molalidad:

1.78 g Fe(OH)2 1 mol Fe(OH)2 1 000 g agua


∗ ∗ = 0.198 molal
100 g de agua 90 g Fe(OH)2 1 kg de agua

a) Las fracciones molares serán:


1.78 g Fe(OH)2
n Fe(OH) 2 = = 0.0198 mol
90 g/mol
100 g H 2 O
n H 2O = = 5.56 mol
18 g/mol

COMPENDIOS DE QUÍMICA GENERAL PARA ESTUDIANTES DE CIENCIAS E INGENIERÍA


SOLUCIONES Y PROPIEDADES COLIGATIVAS 91
0.0198
χ Fe(OH) 2 = = 0.0035
5.5798

3.10 Se tiene una solución de dicromáto de potasio al 1%, considerando solución ideal,
calcular el volumen de esta solución que se necesita para preparar 250 cm3 de solución 0.1
normal de dicromato al actuar como oxidante.

Solución: Para resolver éste problema se debe tomar en cuenta que el dicromáto actúa como
oxidante, esto implica tener cuidado en el cálculo del equivalente gramo ya que se trata de una
reacción de oxidación y reducción.

El dicromáto de potasio al entrar en contacto con el agua se disocia según:


K2Cr2O7= = 2 K+ + Cr2O7=

En solución el dicromáto se reduce de acuerdo a:

6e- + 14H+ + Cr2O7=⇒ 2Cr+3 + 7H2O0

En consecuencia el equivalente gramo del dicromáto de potasio es:

1 mol K 2Cr2O 7
eq − g K 2Cr2 O 7 =
6
6 eq − g K 2Cr2 O 7 = 1 mol K 2Cr2 O 7

1 g K 2 Cr2 O 7 1 mol K 2 Cr2 O 7 6 eq − gK 2 Cr2 O 7 1 g de solución


∗ ∗ ∗ ∗
100 g de solución 294 g K 2 Cr2 O 7 1 mol K 2 Cr2 O 7 1 cm3 de solución

1000 cm 3 de solución
= 0.204 N
1 litro de solución

Al igualar los equivalentes de las dos soluciones se tiene:

N1V1 = N 2 V2

250 cm3 ∗ 0.1 N


V2 = = 122.55 cm 3
0.204 N

3.11 Se queman 80 litros de fosfamina medidos a 18 °C y 756 mmHg. El ácido fosfórico


formado se disuelve en agua y se obtiene un litro de disolución. Calcular la normalidad de la
disolución ácida.

Solución: El proceso de formación de una disolución de H3PO4 viene a partir de la siguiente


ecuación química:

PH3 + 2O2 ⇒ H3PO4

A partir de esta ecuación química se calcula estequiométricamente la cantidad de sustancia de


H3PO4

COMPENDIOS DE QUÍMICA GENERAL PARA ESTUDIANTES DE CIENCIAS E INGENIERÍA


92 SOLUCIONES Y PROPIEDADES COLIGATIVAS
Considerando la ecuación de estado:
PV = nRT se tiene:

PV 756 mmHg ∗ 80 
n= = = 3.33 mol NH 3
RT mmHg − 
62.4 ∗ 291 K
K − mol
1 mol H 3PO 4
3.33 mol NH 3 ∗ = 3.33 mol H 3PO 4
1 mol PH 3

La relación entre moles del ácido y los eq-g se obtiene:

1 mol H 3PO 4
1eq − g H 3PO 4 =
3
3eq − g H 3PO 4 = 1 mol H 3PO 4

La normalidad de la solución ácida será:

3.33 mol H 3PO 4 3 Eq − g H 3 PO 4


N= ∗ = 10 N
1 litro de solución 1 mol H 3 PO 4

3.12 Calcular el número de gramos de sulfato ferroso que se oxidarán en una disolución de
esta sal acidificada con ácido sulfúrico, con 24 ml de permanganato de potasio, en solución
0.25 N, considerando como agente oxidante para esta reacción.

Solución: Formulamos e igualamos la ecuación química por el método ión - electrón

FeSO4 + H2SO4 + KMnO4 ⇒ Fe2(SO4)3 + MnO + K2SO4 + H2O

Las semirreacciones son:

2 Fe+2 ⇒ 2 Fe+3 + 2 e- (1)

5 e- + 8H+ + MnO4- ⇒ Mn+2 + 4H2O (2)

multiplicando la primera ecuación por 5 y la ecuación (2) por 5:

10 Fe+2 + 16H+ + 2 MnO4- ⇒ 10 Fe+3 + 2 Mn+2 + 8H2O

La ecuación igualada es

10FeSO4 + 8 H2SO4 + 2 KMnO4 ⇒ 5 Fe2(SO4)3 + 2 MnSO4 + K2SO4 + 8 H2O

Para la determinación del eq – g de KMnO4, debemos considerar los electrones ganados


porque se trata de una reacción de oxidación – reducción.

1 mol KMnO4
1 eq − g KMnO4 =
5
5 eq − g KMnO4 = 1 mol KMnO4

La masa de sulfato ferroso que se oxida es:

COMPENDIOS DE QUÍMICA GENERAL PARA ESTUDIANTES DE CIENCIAS E INGENIERÍA


SOLUCIONES Y PROPIEDADES COLIGATIVAS 93
0.25 Eq − g KMnO 4 1 mol KMnO 4 10 mol FeSO 4
24 ml KMnO 4 ∗ ∗ ∗ ∗
1000 ml solución 5 eq − g KMnO 4 2 mol KMnO 4

152 g FeSO 4
= 0.912 g FeSO 4
1 mol FeSO 4

Propiedades coligativas:

3.13 A 25 ºC la presión de vapor saturado del agua constituye 23.76 mmHg. Hallar a la misma
temperatura, la presión del vapor saturado sobre una disolución acuosa al 5% de carbamida ,
CO(NH2)2.

Solución: Aplicaremos la ley de Rault:


P = Xd.Pº

La fracción molar del disolvente se puede evaluar a partir de la composición que nos dan en el
problema, vale decir:
5 g de carbamida y 95 g de agua

1 mol CO(NH 2 ) 2
5 g CO(NH 2 ) 2 ∗ =
60 g
0,083 mol CO(NH 2 ) 2
1 mol H 2 O
95 gH 2 O ∗ = 5.28 mol H 2 O
18 g H 2 O
5.28
χ H 2O = = 0.985
5.363

P = 0.985 ∗ 23.76 mmHg

3.14 La solución saturada de fenol en agua tiene a 21 ºC la presión de vapor de 18.3 mmHg,
mientras que la del agua pura es 18.65 mmHg. a) Determinar la solubilidad del fenol en agua ,
suponiendo la idealidad. Exprese también el resultado como b) molaridad, c) molalidad y d)
tanto por ciento en masa.

Solución: Datos:
P = 18.3 mmHg P° = 18.65

a) solubilidad = ? b) M = ? c) m = ? d) % masa = ?

a) La solubilidad se define:
g soluto
solubilidad =
100 g de disolvente
Aplicaremos la ley de Rault:

De la ecuación (3.5): P = χP°:

COMPENDIOS DE QUÍMICA GENERAL PARA ESTUDIANTES DE CIENCIAS E INGENIERÍA


94 SOLUCIONES Y PROPIEDADES COLIGATIVAS
md
P = P° Md
md ms
+
Ms Ms
reemplazando datos:
100
18.3 = 18.65 18
100 ms
+
18 94

efectuando operaciones con una máquina electrónica:

ms
= 0.1077
94
m s = 10.12 g C 6 H 5OH
Por tanto la solubilidad es:
10.12 g de soluto
solubilidad =
100 g de agua
b) molaridad

Para soluciones ideales se considera la densidad 1 g/ml La masa total de la solución es:
(100 + 10.12) g = 110.12 g

V = 110.12 g / 1 g/ml = 110.12 ml = 0.11 litros

Los moles de fenol: 10.12 g / 94 = 0.108 mol

0108 mol C 6 H 5 OH
M= = 0.98 molar
0.11 litros solución
c) molalidad
0.108 mol C 6 H 5OH
m= = 1.08 molal
0.10 kg H 2 O
d) % masa:
10.12
%C 6 H 5 OH = ∗ 100% = 9.19%C6 H 5 OH
110.12

3.15 Cuando se disuelven 60 g de una mezcla de glucosa (C6H12O6) y sacarosa (C12H22O11) en


200 g de agua, se registra una presión de vapor en la disolución de 23.28 mmHg a 25 ºC. La
presión de vapor del agua pura a dicha temperatura es 23.76 mmHg. Determinar la
composición porcentual en masa en la mezcla de glucosa y sacarosa.

Solución Datos:
60 g de mezcla de C6H12O6 y C12H22O11
sean :
C6H12O6 = x y C12H22O11 = y en 200 g de H2O.

P = 23.28 mmHg Pº = 23.76 mmHg

Aplicando la ley de Rault:


P = PºXdisolvente;
COMPENDIOS DE QUÍMICA GENERAL PARA ESTUDIANTES DE CIENCIAS E INGENIERÍA
SOLUCIONES Y PROPIEDADES COLIGATIVAS 95

despejando Xd:
P 23.28mmHg
Xd = = = 0.98
P º 23.76mmHg
por definición de fracción molar:
n H 2O
X disolvente =
n H 2O + n soluto
mH 2O
200
MH2O 18
X disolvente = = = 0.98
mH2O m soluto 200 60
+ +
MH2O M soluto 18 M

g
reemplazando datos: M = 264.63
mol
Considerando la ecuación:
nx + ny = nT

x y 60
+ = (1)
180 342 264.63

x + y = 60 (2)

Resolviendo el sistema de ecuaciones se determina:


x = 19.6 g %C6H12O6 = 32.67%

y = 40.4 g %C12H22O11 = 67.33%

3.16 Calcular la cantidad de C2H6O2 que se debe añadir a 1 kg de etanol para reducir su
presión de vapor en 9.5 mmHg a 35 ºC. La presión de vapor del etanol puro a esta temperatura
es 100 mmHg.

Solución: La presión de vapor de la solución es:


(100 - 9.5) mmHg = 90.5 mmHg
Considerando la ley de Rault:
P = PºXdisolvente;

P 90.5 mmHg
Xd = = = 0.905
Pº 100 mmHg
por definición de fracción molar:
n H 2O
X disolvente =
n H 2O + n soluto
m C 2 H 5 OH
1000
M C 2 H 5 OH 46
X disolvente = = = 0.905
m C 2 H 5 OH m soluto 1000 m
+ +
M C 2 H 5 OH M soluto 46 62
ms
= 2.282
62
COMPENDIOS DE QUÍMICA GENERAL PARA ESTUDIANTES DE CIENCIAS E INGENIERÍA
96 SOLUCIONES Y PROPIEDADES COLIGATIVAS
ms = 141.49 g C2H6O2

3.17 A 30 ºC, la presión de vapor del éter dietílico es de 646 mmHg y de la acetona pura de
283 mmHg. Calcule la composición molar de una mezcla cuya presión sea de 460 mmHg,
suponiendo la idealidad.

Solución: Puesto que ambas sustancias son volátiles (A = éter dietílico; B = acetona) se
cumple que:
Ptotal = PA + PB = XA.PAº + XB.PBº

460 = χA∗646 + χB∗283 (1)

χA + χB = 1 (2)

χA = 1 - χB (3)

(3) reemplazando en (1):


460 = 646 (1 - χB ) + 383 χB
Despejando la fracción molar de la acetona

χB = 0.51; χA = 0.49

La composición molar es
%nacetona = 0.51∗100 = 51.0%

%néter = 0.49∗100 = 49.0%

3.18 En un compuesto orgánico recientemente sintetizado, al realizar el análisis químico se ha


encontrado que contiene 63.2% de carbono, 8.8% de hidrógeno y el resto oxígeno. Una
disolución de 0.0702 gramos de este compuesto en 0.804 g de alcanfor deprime el punto de
solidificación en 15.3 °C. ¿Cuál es la fórmula molecular del compuesto desconocido? La
constante crioscópica del alcanfor es igual a 40.

Solución: Determinación de la fórmula empírica:

63.2 5.266
C: = 5.266 C: =3
12 1.75
8.8 8.8
H: = 8.8 H: =5
1 1.75
18 1.75
O: = 1.75 O: =1
16 1.75

La fórmula empírica es: C3H5O (M = 57)

Para determinar la fórmula molecular debemos calcular el peso molecular de la sustancia a


partir de la expresión del descenso crioscópico.

∆Tc = Kc∗m

m = 15.3 °C / 40 °C/mol = 0.3825

COMPENDIOS DE QUÍMICA GENERAL PARA ESTUDIANTES DE CIENCIAS E INGENIERÍA


SOLUCIONES Y PROPIEDADES COLIGATIVAS 97
El peso molecular calculamos a partir de la expresión (3.8)

a • 1000
m=
A•M

a ∗ 1000 0.0702 ∗ 1000


M= = = 228.27
A∗m 0.804 ∗ 0.3825
En la expresión: (C3H5O)n
228.27
n= =4
57
La fórmula molecular es: C12H20O4

3.19 Hallar la concentración en gramos por litro de una solución de glucosa C6H12O6 que a 25
°C es isosmótica con una disolución de úrea CO(NH2)2 a 12 °C la cual contiene 3 g de úrea en
un volumen de 150 cm3.

Solución El término isosmótico implica la misma presión osmótica pero a diferentes


temperaturas, así que se puede escribir la expresión de presión osmótica para ambas
sustancias e igualarlas.

π = cRT

Para la solución de glucosa se tiene:

π1 = c1RT1 (1)
Para la solución de úrea:
π2 = c2RT2 (2)

T1 = 25 + 273 = 298 K
T2 = 12 + 273 = 285 K
La concentración C2, de la úrea es:

3 g úrea 1 mol de úrea 1000 ml solución


C2 = ∗ ∗ = 0.333 molar
150 ml solución 60g úrea 1 litro solución

Igualando las ecuaciones (1) y (2)


c1RT1 = c2RT2

Despejando C1:
T2 285
c1 = ∗ c2 = ∗ 0.333molar = 0.318molar
T1 298
moles C 6 H12 O 6 180 g C 6 H12 O 6
0.318 ∗ = 57.24 g/
1 litro de solución 1mol C 6 H12 O 6

3.20 El bromuro de etileno, C2H4Br2, y el 1,2 dibromopropano C3H6Br2, forman una serie de
disoluciones ideales en todas sus concentraciones. A 85 ºC, la presión de estos dos líquidos
puros es 173 y 127 torr respectivamente. a) Si se disuelven 10 g de bromuro de etileno en 80 g
de 1,2 dibromopropano, calcúlese la presión de cada componente y la presión total de la
solución a 85 ºC. b) Calcúlese la fracción molar del bromuro de etileno en el vapor en el
equilibrio con la solución anterior. c) Cuál será la fracción molar del bromuro de etileno en
una solución a 85 ºC en equilibrio con una mezcla de 50:50 moles en el vapor?
COMPENDIOS DE QUÍMICA GENERAL PARA ESTUDIANTES DE CIENCIAS E INGENIERÍA
98 SOLUCIONES Y PROPIEDADES COLIGATIVAS

Solución: Para calcular la presión de cada componente se debe considerar la ley de Rault:

PC2H4Br2 = χ C2H4Br2 ∗P° C2H4Br2


PC2H4Br2 = χ C3H6Br2 ∗P° C3H6Br2

P° C2H4Br2 = 173 mmHg


P° C3H6Br2 = 127 mmHg
Las fracciones molares son:
1 mol C 2 H 4 Br2
10 g C 2 H 4 Br2 ∗ = 0.053 mol C 2 H 4 Br2
187.84 g C 2 H 4 Br2
1 mol C 3 H 6 Br2
80 g C 3 H 6 Br2 ∗ = 0.396 mol C 2 H 4 Br2
201.84 g C 3 H 6 Br2

0.053
χ C 2 H 4 Br2 = = 0.118
0.449
χ C3 H 6 Br2 = 1 − 0.118 = 0.882
Las presiones de vapor parciales son:

PC2H4Br2 = 0.118 ∗ 173 mmHg = 20.414 mmHg


PC3H6Br2 = 0.882 ∗ 127 mmHg =112.014 mmHg

La presión de vapor de la solución es:


Ptotal = PC2H4Br2 + PC3H6Br2 = (20.414 + 112.014) mmHg

Ptotal = 132.428 mmHg

Puesto que se trata de soluciones ideales, su comportamiento puede mostrarse en un diagrama


fracción molar vs presión de vapor de ambas soluciones:

Del diagrama realizada a escala se puede dar una


Presión de vapor [mmHg]

interpretación del comportamiento de estas


soluciones.

Por ejemplo se puede dar eventualmente la presión


χ Dibromo propano
total de la solución ideal cuando las fracciones 0.88 0.5
molares de ambas sustancias son iguales, es decir
0.5.

Trace una línea en la fracción molar 0.5 y lea la 173


intersección en presión de vapor esto da 150
aproximadamente 150 mmHg. en la práctica resulta 132
127
muy útil el uso de estos diagramas.

c) Para determinar las fracciones molares de estas


sustancias en fase vapor, debemos recordar un
concepto muy importante de las fracciones molares
en función de las presiones parciales.
0.12 0.5 1.0
χ Bromuro de etileno
COMPENDIOS DE QUÍMICA GENERAL PARA ESTUDIANTES DE CIENCIAS E INGENIERÍA
SOLUCIONES Y PROPIEDADES COLIGATIVAS 99
P
χA = A
Ptotal

Entonces la fracción molar del bromuro de etileno en la fase vapor es:

20.414
χ C 2 H 4 Br2 = = 0.154
132.428

d) La fracción molar del bromuro de etileno en la fase líquida cuando las fracciones molares
en fase gaseosa son iguales será:
Sean: A = C2H4Br2 y B = C3H6Br2

PA PB
En fase gaseosa se cumple: X A' = y X B' = (1)
PT PT

Además: PA = PA° X A PB = PB° X B (2)

Donde X’A y X’B son las fracciones molares en fase gaseosa y XA y XB, son las fracciones
molares en solución

Despejando PA y PB de (1)

PA = PT X A' PB = PT X B' (3)

Igualando ahora las ecuaciones (2) y (3)


PA° X A = PT X A'
PB° X B = PT X B'

Dividiendo estas ecuaciones y tomando en cuenta que X’A = X’B = 0.5

PA° X A = PB° X B
X A PB° 127torr
= = = 0.734
X B PA° 173torr
Considerando además que:
XA + XB = 1

Y resolviendo el sistema de ecuaciones, encontramos que las fracciones molares en fase líquida son:

XA = 0.42

XB = 0.58

3.21 El benceno congela a 5.5 °C y hierve a 80.2 °C. Los calores latentes de fusión y
ebullición del benceno son, respectivamente, 30.3 y 94.2 cal/g. Calcular las constantes
molales de los puntos de a) congelación y de b) ebullición del benceno. Los valores
experimentales so 5.12 y 2.67 °C/molal, respectivamente

Solución: Las constantes molales dependen del calor latente de ebullición y fusión
respectivamente, y aproximadamente se puede calcular considerando la siguiente ecuación:
COMPENDIOS DE QUÍMICA GENERAL PARA ESTUDIANTES DE CIENCIAS E INGENIERÍA
100 SOLUCIONES Y PROPIEDADES COLIGATIVAS

a)
cal
1.987 ∗ ( 278.5K ) 2
RTc2 K − mol
Kc = = = 5.09 K/mol
1000Δ00 1000 g ∗ 30.3 cal/g

b)
cal
1.987 ∗ ( 353.2K ) 2
RTe2 K − mol
Ke = = = 2.63 K/mol
1000Δ00 1000 g ∗ 94.2 cal/g

3.22 En un aparato para helados de tipo casero se abate el punto de congelación de un baño
de agua que rodea el helado disolviendo NaCl para obtener una solución salina. Se observa
que una solución salina al 15% se congela a – 10.880 °C. ¿Cuál es el factor de van’t Hoff, i,
para esta solución?
Solución: Datos

Tf(agua) = 0 °C Tf(solución) = -10.88 °C

Kc = 1.86 °C/m

15 g NaCl 1 mol NaCl 1000 g H 2 O


m= ∗ ∗ = m = 3.02 molal
85 g H 2 O 58.5 g NaCl 1 kg H 2 O

Consideremos la ecuación ∆Tc = iKcm y despejando i, se tiene:

ΔTc
i=
Kc ∗ m

0°C − (−10.88°C )
i= = 1.94
°C
1.86 ∗ 3.02m
m

3.23 El cloruro de cesio se disuelve en agua según la siguiente reacción:

CsCl ⇒ Cs+ + Cl-

Una solución 0.121 m de CsCl se congela a – 0.403. Calcule i y la disociación porcentual de


CsCl en esta disolución.

Solución:

Considerando la ecuación ∆Tc = iKcm y despejando i, se tiene:

ΔTc
i=
Kc ∗ m
donde: ∆Tc = 0 °C – (- 0.403 °C) = 0.403 °C

COMPENDIOS DE QUÍMICA GENERAL PARA ESTUDIANTES DE CIENCIAS E INGENIERÍA


SOLUCIONES Y PROPIEDADES COLIGATIVAS 101
El factor de Van’t Hoff será:
0.403°C
i= = 1.7906
°C
1.86 ∗ 0.121m
m

La disociación porcentual se calcula considerando la ecuación:

i −1
α=
υ −1

los iones formados, como se observa en la ecuación de disociación es: ν = 2, reemplazando en


la anterior expresión:
1.7906 − 1
α= = 0.7906
2 −1
La disociación porcentual es:
α = 0.7906 ∗ 100% = 79.06%

3.24 El compuesto complejo, K3[Fe(CN)6], se disuelve en agua según la siguiente reacción:

K3[Fe(CN)6] ⇒ 3K+ + [Fe(CN)6-3]

Una solución 0.126 m de K3[Fe(CN)6] se congela a – 0.649 °C. Calcule el factor de Vant’
Hoff y la disociación porcentual de K3[Fe(CN)6] en esta solución.

Solución: Considerando la ecuación ∆Tc = iKcm y despejando i, se tiene:

ΔTc
i=
Kc ∗ m
donde ∆Tc = 0 °C – (- 0.649 °C) = 0.649 °C

El factor de Van’t Hoff será:


0.649 °C
i= = 2.769
°C
1.86 ∗ 0.126 m
m
La disociación porcentual se calcula considerando la ecuación:

i −1
α=
υ −1

los iones formados, como se observa en la ecuación de disociación es: ν = 4, reemplazando en


la anterior expresión:

2.769 − 1
α= = 0.5897
4 −1

La disociación porcentual es:

α = 0.5897 ∗ 100% = 58.97%

COMPENDIOS DE QUÍMICA GENERAL PARA ESTUDIANTES DE CIENCIAS E INGENIERÍA


102 SOLUCIONES Y PROPIEDADES COLIGATIVAS
3.25 Al mezclar 17.6 kg del anticongelante comercial (etilenglicol, C2H6O2) con 6 galones de
agua, se abate su punto de congelación a –10.0 °F. Suponga que se desea lograr el mismo
efecto con sacarosa, C12H22O11 (que es una mala idea), en vez de etilenglicol. ¿Cuántos
kilogramos de sacarosa necesitaría disolver? 1 galón = 3.785 litros.

Solución: En principio calcularemos la molalidad del etilenglicol, puesto que hay que abatir el
punto de congelación a – 10.0 °F las molalidades deben ser idénticas, es decir:

17.6 kg C 2 H 6 O 2 1000 g C 2 H 6 O 2 1 mol C 2 H 6 O 2


m= ∗ ∗ = 12.5 molal
3.785  1 kg 1 kg C 2 H 6 O 2 62 g C 2 H 6 O 2
6 gal H 2 O ∗ ∗
1 gal 1 

moles C12 H 22 O11 342 g C12 H 22 O11 1 kg C12 H 22 O11 1 kg H 2 O 3.785 H 2 O


12.5 ∗ ∗ ∗ ∗ ∗
1 kg H 2 O 1 mol C12 H 22 O11 1 000 g C12 H 22 O11 1 H 2 O 1 gal H 2 O
6gal = 97.09 kg C12 H 22 O11

a partir de la molalidad ha sido posible determinar la masa en kg de sacarosa para abatir la


temperatura de congelación, esto es se necesitan 97.09 kg de sacarosa.

PROBLEMAS PROPUESTOS

(Soluciones)

3.1 a) Calcular la cantidad de sosa caústica y de agua que se necesita para


preparar 5 litros de una solución al 20%, cuya densidad es 1.219 g/ml, b)
Cual es la normalidad de está disolución?, c) Cuál es la molalidad?
Rpta.- 1219 g NaOH y 4876 g H2O; 6.095 N

3.2 Calcular el volumen de disolución al 12% en peso de CuSO4, que podrá prepararse con 1
kg de sulfato cúprico cristalizado, (CuSO4∗5H2O). La densidad de la solución es 1.131 g/ml.
Rpta.- 4.71 litros

3.3 Determinar la masa de sulfato magnésico heptahidratado, (MgSO4∗7H2O), que debe


añadirse a 1.5  de agua para obtener una disolución al 20% en masa de sulfato anhídro.

3.3 Se disuelven X g de MgCl2 en agua formando 2 litros de solución, observándose una


concentración de 24 mg de Mg+2 en cada cm3 de solución. Hallar el valor de X.
Rpta. 190

COMPENDIOS DE QUÍMICA GENERAL PARA ESTUDIANTES DE CIENCIAS E INGENIERÍA


SOLUCIONES Y PROPIEDADES COLIGATIVAS 103
3.5 Se disuelven 130 g de una base de metal monovalente desconocida en agua formándose 1
litro de solución. Hallar su normalidad si se sabe que 0.78 g de la base produce 0.03 moles de
(OH)-1.
Rpta. 5 normal

3.7 Calcular el volumen de una disolución de ácido sulfúrico de densidad 1.827 g/ml y
92.77% de H2SO4 que se necesita para preparar 10 litros de ácido sulfúrico 3 normal.
Rpta.- 868.2 ml

3.8 Determinar el volumen de una disolución 2 normal de sulfato cúprico que se necesita para
preparar 10 g de óxido cúprico previa precipitación del cobre como carbonato y calcinación
posterior de éste a óxido. Considerar las siguientes reacciones:

Na2CO3 + CuSO4 → CuCO3 + Na2SO4

CuCO3 + calor → CuO + CO2

3.9 ¿Qué volumen de ácido nítrico diluido, de densidad 1.11 g/ml y al 19% en masa de HNO3,
contiene 10 g HNO3 ?

3.10 Calcúlese la molalidad de una solución que contiene a) 0.65 moles de glucosa, en 250 g
de agua, b) 45 g de glucosa en 1 kg de agua, c) 18 g de glucosa en 1 libra de agua.
Rpta.- a) 2.6 m, b) 0.25 m, c) 0.30 m

3.11 Una solución acuosa etiquetada muestra una concentración del 35% en masa de HClO4,
una densidad de 1.251 g/cc. ¿Cuál es la concentración molar y la molalidad de la solución?
Rpta.- 4.36 M; 5.36 m
3.12 El agua y el metanol son miscibles en todas las proporciones, si se mezclan 16 g de
metanol con 27 g de agua. ¿Cuál es la fracción molar del metanol en la solución?

3.13 Realice las siguientes conversiones siguientes:

a) Ca(OH)2 0.1 N en molaridad


b) 20 mg CuSO3/ml en molaridad y normalidad
c) 2.4 mg CaCl2/ml en molaridad y normalidad
d) 1.2 molal de nitrato de plata en tanto por ciento en masa
e) 2 N de H3PO4 en molaridad

3.14 Una reacción requiere 3.4 milimoles de Na3PO4. ¿Cuántos mililitros de una disolución
1.8 N se usarían?

3.15 Se mezclan 150 cc de K2SO4 3 M con 80 cc de NaNO3 2M. ¿Cuál es la concentración de


cada sal en la disolución al final?

3.16 ¿Cuántos mililitros de H2SO4 3.0 M se necesitan para neutralizar 200 ml de 0.34 N de
Ca(OH)2 ? ¿Cuántos miliequivalentes de CaSO4 se formaran?

3.17 El ácido clorhídrico concentrado tiene una concentración de 37% en masa de HCl y tiene
una densidad relativa de 1.184. Hallar la molaridad, normalidad y molalidad.
COMPENDIOS DE QUÍMICA GENERAL PARA ESTUDIANTES DE CIENCIAS E INGENIERÍA
104 SOLUCIONES Y PROPIEDADES COLIGATIVAS

3.18 Se titularon exactamente 50 cc de una disolución de Na2CO3 con 65.8 cc de HCl 3.0 N de
acuerdo a:
Na2CO3 + 2HCl → NaCl + CO2 + H2O

Si la densidad de la solución de Na2CO3 es 1.25 g/cc. ¿Qué porcentaje de Na2CO3 en masa


contiene?
Rpta.- 16.7%

3.19 a) Que volumen de K2Cr2O7 0.40 N se necesita para liberar el cloro de 1.2 g de NaCl en
una solución acidifícada con H2SO4 ?

Cr2O7= + Cl- + H+→ Cr+3 + Cl2 + H2O

b) ¿Cuántos gramos de cloro gaseoso se liberan?


Rpta.- 51 ml; 0.73 g

3.20 ¿Qué volumen de una disolución de perclorato férrico 0.5 N se necesitan para preparar
25 g de cloruro férrico?, en una solución básica de hidróxido plumboso, considerando la
siguiente reacción química.

Pb(OH)2 + Fe(ClO4)3 → FeCl3 + Pb3O4 + H2O

3.21 Si 600 cc de una solución 1.1 N de HCl son diluidos hasta formar exactamente la
solución 1 normal. ¿Qué cantidad de agua ha sido adicionada?

3.22 Una solución de NaOH es 0.9 N. ¿Qué cantidad de la misma será necesaria para preparar
un litro de solución 0.1 N?

3.23 40 ml de solución 0.5 N de HCl se mezclan con 30 cc de solución normal de H 2SO4.


¿Cuántos ml de solución 0.333 N de NaOH serán necesarios para neutralizar la mezcla de
ácidos?

3.24 Una muestra de un hidróxido alcalino puro (NaOH o KOH) es disuelta en agua y
requiere 50 cc de solución ácida 0.5 N para su neutralización. ¿De cuál de las dos bases
indicadas era la muestra?

3.25 Una muestra de 1.5 g de un metal puro ha necesitado 45.9 cc de solución normal de HCl
para ser disuelta. El metal es bivalente; calcular su peso atómico aproximado.

3.26 Cuantos gramos de hierro se disuelven en medio litro de solución 0.1 N de HCl?

3.27 10 g de una solución acuosa que contiene solamente Na2SO4 y H2SO4 se valora con
solución 0.5 normal de NaOH, de la cual se gastan 24.26 cc para la neutralización; la solución
resultante es tratada con BaCl2 en exceso y se obtiene 3.1526 g de BaSO4. Calcular el
porcentaje de a) H2SO4 y b) Na2SO4 en la solución original.

COMPENDIOS DE QUÍMICA GENERAL PARA ESTUDIANTES DE CIENCIAS E INGENIERÍA


SOLUCIONES Y PROPIEDADES COLIGATIVAS 105
3.28 Si 51 cc de solución 0.1 N de HCl son tratados con 49 cc de solución 0.1 N de NaOH,
calcular: a) la normalidad en ácido de la solución resultante; b) el número de gramos de NaCl
que dejará la solución al ser evaporada a sequedad.

3.29 La solución de HCl de peso específico relativo 1.9545 contiene 10.94% de HCl en masa,
¿Cuántos ml son necesarios para preparar dos litros de solución 2 N?

3.30 Cuántos litros de gas NH3 seco medido a 27 ºC y 733 mmHg , serán necesarios para
preparar dos litros de solución exactamente ½ N de NH4OH.

3.31 25 cc de solución 0.5 N de H2SO4 son agregados a 40 cc de solución 0.25 N de NaOH.


¿Cuál está en exceso y en que cantidad?. Expresar la normalidad de la solución resultante en
función de a) el ácido o la base en exceso; b) el ión sulfato.

3.32 Calcular la pureza en tanto por ciento de una muestra de H2C2O4∗2H2O partiendo del
dato de que 1.00 g de la misma es neutralizado por 31.4 cc de solución básica 0.5 N.

3.33 Si 1 g de H2C2O4∗2H2O puro neutraliza 33.0 cc de una solución de KOH, calcular la


concentración de ésta en: a) gramos de KOH por litro; b) normalidad como bases; c)
molaridad.

3.34 La solución de NH4OH de peso específico relativo 0.954 contiene 11.64% de NH3.
Calcular su normalidad como base.

3.35 Masa iguales de NaOH y KOH son disueltos separadamente en la misma cantidad de
agua. ¿Cuál es la relación de sus normalidades?.

3.36 Calcular la normalidad de una solución de K2Cr2O7 sabiendo que 48 cc de la misma


oxidan 1.375 g de FeSO4∗7H2O puro.

3.37 Una solución de H2SO4 de peso específico relativo 1.160 contiene 22.25% en masa de
H2SO4. Calcular: a) su normalidad; b) su molaridad, c) su molalidad, d) su fracción molar.

3.38 Leemos en un libro de prácticas de química: Si se miden 40 ml de HCl concentrado


(densidad 1.195 g/ml, 38.72% HCl) y se diluye hasta formar un litro de solución, ésta resulta
aproximadamente 0.5 N. Háganse los cálculos para ver si esto es verdad.
Rpta. 0.507 N

3.39 Calcular la normalidad de una solución concentrada de H2SO4 según los datos siguientes,
densidad 1.8068 g/ml y 87.81% en masa

3.40 Se desea preparar exactamente 10 litros de solución 0.750 N de NaOH. ¿Cuántos


mililitros de solución de sosa cáustica de densidad 1.160 y 14.45% en masa de NaOH deben
emplearse?.

3.41 Para preparar 10 litros exactos de solución 0.111 N de KOH, disponemos de 40.22 g de
KOH puro. ¿Cuántos ml de solución de potasa cáustica, de peso específico 1.3010 del 31% en
masa de KOH son necesarios para completar la solución?

COMPENDIOS DE QUÍMICA GENERAL PARA ESTUDIANTES DE CIENCIAS E INGENIERÍA


106 SOLUCIONES Y PROPIEDADES COLIGATIVAS
3.42 Se disuelve 350 g de cloruro de cinc anhidro (densidad relativa 2.91) en 650 g de agua,
se obtiene una disolución cuyo volumen total, a 20 ºC es de 740 ml. Calcular: a) la molaridad,
b) la normalidad, c) la molalidad, d) las fracciones molares, e) el tanto por ciento en masa
Rpta. 3.47 M, 6.94 N, 3.95 m, 0.0665, 35% ZnCl

(Propiedades Coligativas)

3.43 A la temperatura de 65 ºC hallar la presión de vapor sobre una disolución que contiene
13.68 g de sacarosa en 90 g de agua, si la presión de vapor saturado sobre el agua a la misma
temperatura es igual a 25.0 kPa (187.5 mm Hg).

3.44 ¿A que es igual la presión de vapor saturado sobre una disolución al 10% de carbamida
CO(NH2)2 a 100 ºC?

3.45 A la temperatura de 315 K, la presión de vapor saturado sobre el agua es igual a 8.2 kPa
(61.5 mmHg). ¿Cuanto disminuirá la presión de vapor a la temperatura señalada, si en 540 g
de agua se disuelven 36 g de glucosa?

3.46 A 293 K la presión de vapor saturado sobre el agua es igual a 2.34 kPa (17.53 mmHg).
¿Cuántos gramos de glicerina C3H5(OH)3 se deben disolver en 180 g de agua para disminuir la
presión de vapor en 133.3 Pa?

3.47 ¿Cuántos grados aumentará el punto de ebullición del agua si en 100 g de ésta se
disuelven 9 g de glucosa?

3.48 ¿A que temperatura, aproximadamente, hervirá la disolución al 50% en masa de


sacarosa?

3.49 ¿A que temperatura, aproximadamente, cristalizará la disolución al 40% en masa de


alcohol etílico?.

3.50 ¿Cuántos gramos de sacarosa se deben disolver en 100 g de agua para: a) disminuir la
temperatura de cristalización en 1 grado; b) aumentar la temperatura de ebullición en 1 grado?

3.51. ¿En que relación deben encontrarse las masas de agua y de alcohol etílico para que al
mezclarlos, se obtenga una disolución que se cristalice a –20 ºC?.

3.52 En el radiador de automóvil vertieron 9 litros de agua y añadieron 2 litros de alcohol


metílico (ρ = 0,8 g/ml. Hecho esto, ¿a que temperatura mínima se puede dejar el coche al aire
libre sin temer que el agua en el radiador se congele?

3.53 Al disolver 5.0 g de sustancia en 200 g de agua se obtiene una disolución no conductora
de corriente, la cual se cristaliza a -1.45 ºC. Determine el peso molecular del soluto.

3.54 Al disolver 13 g de no electrolito en 400 g de éter dietílico (C2H5)2O, la temperatura de


ebullición se elevó en 0.453 K. Determinar el peso molecular del soluto.

3.55 En 60 g de benceno están disueltos 2.09 g de cierta sustancia cuya composición


elemental en masa es como sigue: 50.69% de C; 4.23% de H y 45.08% de O. La disolución se
COMPENDIOS DE QUÍMICA GENERAL PARA ESTUDIANTES DE CIENCIAS E INGENIERÍA
SOLUCIONES Y PROPIEDADES COLIGATIVAS 107
cristaliza a 4.25 ºC. Establecer la fórmula molecular de la sustancia. El benceno puro se
cristaliza a 5.5 ºC

3.56 Una disolución acuoso-alcohólica que contiene 15% de alcohol desconocido (ρ = 0.97 g/
ml) se cristaliza a -10.26 ºC. Hallar el peso molecular del alcohol desconocido y la presión
osmótica de la disolución a 293 K.

3.57 La temperatura de ebullición de la disolución acuosa de sacarosa es igual a 101.4 ºC.


Calcular la concentración molal y el tanto por ciento en masa de la sacarosa en la disolución.
¿A que temperatura se congela esta disolución?

3.58 ¿Cuántos gramos de glucosa C6H12O6 deben encontrarse en 0.5 litros de disolución para
que su presión osmótica (a igual temperatura) sea la misma que la de una disolución, 1 litro de
la cual contiene 9.2 g de glicerina C3H5(OH)3?

3.59 A 25 ºC la presión osmótica de la disolución cuyos 200 ml contiene 2.80 g de compuesto


macromolecular es igual a 0.70 kPa. Hallar el peso molecular del soluto.

3.60 A la temperatura de 20 ºC y 1 litro de disolución de un no electrolito cuya presión


osmótica es de 243.4 kPa se mezcla con 3 litros de disolución de no electrolito cuya presión
osmótica es igual a 486.8 kPa. Determinar la presión osmótica de la disolución mezclada

3.61 Una disolución en cuyos 100 ml se encuentran 2.3 g de cierta sustancia presenta, a 298
K, una presión osmótica igual a 618.5 kPa. Determinar el peso molecular de la sustancia

3.62 Un ml de disolución contienen 1018 moléculas de no electrolito disuelto. Calcular la


presión osmótica de la disolución a 298 K.

3.63 La presión de vapor del agua pura a 25 ºC es de 23.76 mmHg, ¿Cuál es la presión de
vapor de una disolución que contiene 12 g de glucuosa, en 50 g de agua ?

3.64 Se disuelven 24.63 g de glucosa en 150 g de agua. A 23 ºC la presión de vapor de la


disolución es de 20.73 mmHg y la del agua pura es de 21.07 mmHg. ¿Qué peso molecular
tiene la glucosa?

3.65 A 30 ºC, la presión de vapor del eter dietílico es de 646 mmHg y la de la acetona pura de
283 mmHg.. Calcule la composición de una mezcla cuya presión sea 460 mmHg, suponiendo
la idealidad.

3.66 A 11 ºC las presiones de vapor del clorobenceno, C6H5Cl y del bromobenceno C6H5Br
son, respectivamente de 400 y 200 mmHg. Determinar la presión de vapor a esta temperatura
de una mezcla líquida supuesta ideal, formada por un 30% de C6H5Cl y 70% de C6H5Br, en
masa.

3.67 Suponga que se disuelven 5.0 gramos de una mezcla de naftaleno y antraceno, en 300
gramos de benceno. Se observa que la disolución se congela a 4.85 ºC. Encuentre la
composición porcentual (en masa) de la mezcla.

COMPENDIOS DE QUÍMICA GENERAL PARA ESTUDIANTES DE CIENCIAS E INGENIERÍA


108 SOLUCIONES Y PROPIEDADES COLIGATIVAS
3.68 Al mezclar 17.6 kg del anticongelante comercial (etilenglicol, C2H6O2) con 6 galones de
agua, se abate su punto de congelación a –10 ºF. Suponga que se desea lograr el mismo efecto
en sacarosa, C12H22O11 ( que es una mala idea), en vez de etilenglicol. ¿Cuántos kg de sacarosa
necesitaría disolver? ( 1 galón = 3.785  )

3.69 El calor latente de fusión del cloruro mercúrico (HgCl2) es de 16.9 cal/g en su punto de
fusión de 265 °C. Al colocar en 50 g de aquél, que actúa como disolvente, 0.849 g de cloruro
mercuroso que actúa como soluto, el punto de fusión de la solución desciende en 1.24 °C.
Estimar el peso molecular del cloruro mercuroso y su fórmula molecular.
Rpta.- 236.1; Hg2Cl2

3.70 Una disolución de 0.52 g de cloruro potásico en 83.6 g de agua congela a - 0.291 °C.
Calcular el factor de van’t Hoff, el coeficiente osmótico y el grado de disociación aparente del
KCl. Kc(H2O) = 1.86 °C/mol.
Rpta. i = 1.876; g = 0.938; α = 0.876

3.71 El punto de ebullición de una disolución de 3.41 g de cloruro cálcico en 100 g de agua es
100.21 °C. Calcular el factor de van’t Hoff, el coeficiente osmótico y el grado de disociación
aparente del cloruro de bario. Ke(H2O) = 0.52 °C/mol
Rpta. i = 2.466; g = 0.822; α = 0.733

3.72 Una disolución de cloruro de cinc al 1%, y de densidad prácticamente la unidad, congela
a – 0.28 °C. Calcular el grado de disociación aparente del cloruro de cinc, y a partir de él, la
concentración de los iones en la disolución.
Rpta. α = 0.516; [Zn+2] = 0.0375 M

3.73 Calcular la presión de vapor a 100 °C de una disolución de cloruro de sodio al 10%,
suponiendo que el grado de disociación aparente de la sal sea del 90%.
Rpta. 713.6 mmHg

3.74 Una disolución de cloruro potásico que contiene 1 g de sal por litro ejerce , a 14 °C, una
presión osmótica de 456 mmHg. Calcular el valor del coeficiente osmótico y el grado de
disociación aparente del cloruro potásico.
Rpta. g = 0.951; α = 90.1%

3.75 Calcular la presión osmótica a 18 °C de una solución de cloruro sódico que contiene 2.5
g de sal en 100 cm3, si el factor de van’ Hoff es igual a 1.83.
Rpta. 18.68 atm

3.76 Un cloruro hidratado de calcio contiene 27.21% de Ca y 28.30% de Cl. La disolución


formada al disolver 0.364 g del compuesto en 50 g de agua empieza a congelar a – 0.29 °C. a)
Escribir la fórmula racional del compuesto, b) determine el factor de van’t Hoff. Kc(H2O) =
1.86 °C/mol
Rpta. CaCl2H2O; i = 3.12

3.77 Una solución acuosa con 5.00% en peso de ácido sulfúrico, tiene un punto de
congelación de – 2.17 °C. Calcular:
a) El factor de Van’t Hoff.

COMPENDIOS DE QUÍMICA GENERAL PARA ESTUDIANTES DE CIENCIAS E INGENIERÍA


SOLUCIONES Y PROPIEDADES COLIGATIVAS 109
b) El coeficiente osmótico de la solución
Rpta.- 2.17; 0.723

3.78 Una solución acuosa 0.100 molal de ácido acético, presenta disociado el CH 3COOH en
1.35%. Calcular:
a) El punto de congelación de la solución
b) La presión osmótica de la solución
Rpta.- -0.188 °C; 2.48 atm

3.79 Se ha disuelto 160 g de ácido oxálico en 1000 g de agua, resultando que la solución
hierve a 102.41 °C, a la presión de 1 atm. Calcular el grado de disociación del COOH –
COOH.
Rpta.- 0.82

3.80 El descenso de la presión de vapor producido por un mol de NaCl en 1000 g de agua a 18
°C, es de 0.475 mmHg. La tensión de vapor de agua a 18 °C es de 15.477 mmHg. Calcular:
a) El factor de Van’t Hoff.
b) La presión osmótica
Rpta.- 1.734; 41.38 atm

COMPENDIOS DE QUÍMICA GENERAL PARA ESTUDIANTES DE CIENCIAS E INGENIERÍA

Vous aimerez peut-être aussi